134
Chapter 1 *ABE1-8 Your answer is correct. Use the basic accounting equation to answer these questions. (a) The liabilities of Daley Company are $80,063 and the stockholders' equity is $205,645. What is the amount of Daley Company's total assets? (b) The total assets of Laven Company are $172,724 and its stockholders' equity is $93,034. What is the amount of its total liabilities? (c) The total assets of Peterman Co. are $678,400 and its liabilities are equal to one- fourth of its total assets. What is the amount of Peterman Co.'s stockholders' equity? Assets = Liabilities + Stockholders' Equity (a) $ 285708 $ 80063 $ 205645 (b) $ 172724 $ 79690 $ 93034 (c) $ 678400 $ 169600 $ 508800 Question Attempts: Unlimited AP1-3A (a) Your answer is correct. On June 1 Beardsley Service Company was started with an investment of $27,024 cash. Here are the assets and liabilities of the company on June 30, and the revenues and expenses for the month of June, its first month of operations. Cash $5,424 Notes payable $13,638 Accounts receivable 4,408 Accounts payable 908 Revenue 7,814 Supplies expense 1,021 Supplies 2,389 Gas & oil expense 620 Advertising expense 400 Utilities expense 280 Equipment 29,814 Wage expense 2,214 In June, the company issued no additional common stock, but paid dividends of $2,814. Complete an income statement and a retained earnings statement for the month of June 1

Accounting and Finance

Embed Size (px)

Citation preview

Page 1: Accounting and Finance

Chapter 1

*ABE1-8

Your answer is correct.

Use the basic accounting equation to answer these questions.

(a)  The liabilities of Daley Company are $80,063 and the stockholders' equity is $205,645. What is the amount of Daley Company's total assets?

(b)  The total assets of Laven Company are $172,724 and its stockholders' equity is $93,034. What is the amount of its total liabilities?

(c)  The total assets of Peterman Co. are $678,400 and its liabilities are equal to one-fourth of its total assets. What is the amount of Peterman Co.'s stockholders' equity?

Assets = Liabilities + Stockholders' Equity(a) $ 285708   $ 80063   $ 205645(b) $ 172724   $ 79690   $ 93034(c) $ 678400   $ 169600   $ 508800

Question Attempts: Unlimited

AP1-3A (a)

Your answer is correct.

   On June 1 Beardsley Service Company was started with an investment of $27,024 cash. Here are the assets and liabilities of the company on June 30, and the revenues and expenses for the month of June, its first month of operations.

Cash $5,424 Notes payable $13,638Accounts receivable 4,408 Accounts payable 908Revenue 7,814 Supplies expense 1,021Supplies 2,389 Gas & oil expense 620Advertising expense 400 Utilities expense 280Equipment 29,814 Wage expense 2,214In June, the company issued no additional common stock, but paid dividends of $2,814.

Complete an income statement and a retained earnings statement for the month of June and a balance sheet at June 30, 2012. (List expenses from largest to smallest amount, e.g. 10, 5, 2. List assets in order of liquidity and liabilities from largest to smallest amount, e.g. 10, 5, 2 with notes payable first.)

BEARDSLEY SERVICE CO.Income Statement

1

 

Page 2: Accounting and Finance

Revenues

       $Expenses

       $

      

      

      

      

Total expenses

Net income$

BEARDSLEY SERVICE CO.Retained Earnings Statement

$

Add: 

Less: 

$BEARDSLEY SERVICE CO.

Balance Sheet

Assets

$

2

For the Month Ended June 30, 2012

Revenue 7814

Wage expense 2214

Supplies expense 1021

Gas & oil expense 620

Advertising expense 400

Utilities expense280

4535

3279

For the Month Ended June 30, 2012

Retained earnings, June 1 0

Net income3279

3279

Dividends2814

Retained earnings, June 30465

June 30, 2012

Cash 5424

Accounts receivable 4408

Supplies 2389

Equipment29814

Page 3: Accounting and Finance

Total assets$

 Liabilities and Stockholders' Equity

Liabilities  

       $

      

          Total liabilities$

Stockholders' equity  

      

      

          Total liabilities and stockholders' equity$

Click here if you would like to Show Work for this question

Question Attempts: Unlimited  

Copyright © 2000-2012 by John Wiley & Sons, Inc. or related companies. All rights reserved.

3

42035

Notes payable 13638

Accounts payable908

14546

Common stock 27024

Retained earnings465 27489

42035

User, 08/23/12,
Common Stock is an investment!
Page 4: Accounting and Finance

Chapter 2

AP2-3A

Your answer is correct.

   You are provided with the following information for Merrell Enterprises, effective as of its April 30, 2012, year-end.

Accounts payable $925Accounts receivable 901Building, net of accumulated depreciation 1,537Cash 1,471Common stock 1,302

4

 

Page 5: Accounting and Finance

Cost of goods sold 1,081Current portion of long-term debt 541Depreciation expense 335Dividends paid during the year 416Equipment, net of accumulated depreciation 1,421Income tax expense 165Income taxes payable 226Interest expense 491Inventories 1,058Land 2,301Long-term debt 3,701Prepaid expenses 103Retained earnings, beginning 1,600Revenues 4,801Selling expenses 210Short-term investments 1,310Wages expense 609Wages payable 313(a) Complete income statement and a retained earnings statement for Merrell Enterprises for

the year ended April 30, 2012. (List expenses from largest to smallest amounts, e.g. 10, 5, 1.)

MERRELL ENTERPRISESIncome Statement

Revenues

      $Expenses

       $

      

      

      

      

      

          Total expenses

Net income$

MERRELL ENTERPRISESRetained Earnings Statement

$

Add: 

5

 

For the Year Ended April 30, 2012

Revenues 4801

Cost of goods sold 1081

Wages expense 609

Interest expense 491

Depreciation expense 335

Selling expenses 210

Income tax expense165

2891

1910

 

For the Year Ended April 30, 2012

Retained earnings, May 1 1600

Net income1910

3510

Page 6: Accounting and Finance

Less: 

$(b) Complete the classified balance sheet for Merrell Enterprises as of April 30, 2012. (List assets in

order of liquidity and liabilities from largest to smallest amount e.g. 10, 5, 1.)MERRELL ENTERPRISES

Balance Sheet

AssetsCurrent assets

      $

     

     

     

     

          Total current assets$

Property, plant and equipment

      

      

      

          Total property, plant and equipment

          Total assets$

 Liabilities and Stockholders' Equity

Current liabilities

      $

     

     

     

          Total current liabilities$

6

Dividends416

Retained earnings, April 30

 

3094

April 30, 2012

Cash 1471

Short-term investments 1310

Accounts receivable 901

Inventories 1058

Prepaid expenses103

4843

Land 2301

Building, net of accum. depr. 1537

Equipment, net of accum. depr.1421

5259

10102

Accounts payable 925

Current portion of long-term debt 541

Wages payable 313

Income taxes payable226

2005

Long-term debt3701

User, 08/23/12,
Prepaid Expense is ALWAYS an asset!
Page 7: Accounting and Finance

      Total liabilities

Stockholders' equity

     

     

          Total stockholders' equity

          Total liabilities and stockholders' equity$

Click here if you would like to Show Work for this question

7

5706

Common stock 1302

Retained earnings3094

4396

10102

Page 8: Accounting and Finance

Jump to Navigation Frame Jump to Content Frame

   Your location: Learning Modules › Chapter 1 › Quiz 1 › > Quiz 1 › Assessments › View All Submissions › View Attempt

View Attempt 1 of 1

Title: > Quiz 1

Started: August 22, 2012 8:41 AM

Submitted: August 22, 2012 9:22 AM

Time spent: 00:41:19

Total score: 70/100 = 70% Total score adjusted by 0.0 Maximum possible score: 100

1.

 

The best definition of assets is

Student Response

A. resources belonging to a company having future benefit to the company.

B. collections of resources belonging to the company and the claims on these resourcese.

C. cash owned by the company.

D. owners' investment in the business.

Score: 10/10

2.

8

Page 9: Accounting and Finance

 

Which of the following is not an expense?

Student Response

A. Rent Expense

B. Depreciation Expense

C. Prepaid Expense

D. Salary Expense

Score: 10/10

3.

 

Resources owned by a business are referred to as

Student Response

A. stockholders' equity.

B. liabilities.

C. revenues.

D. assets.

Score: 10/10

4.

  A customer payment

9

Page 10: Accounting and Finance

Student Response

A. decreases expenses.

B. decreases stockholders' equity.

C. increases expenses.

D. increases assets.

Score: 10/10

5.

 

A net loss results when

Student Response

A. Expenses>Liabilities

B. Expenses>Assets

C. Expenses>Revenues

D. Liabilities>Assets

Score: 10/10

6.

  If the retained earnings account increases from the beginning of the year to the end of the year, then

Student Response

10

Page 11: Accounting and Finance

A. additional investments are less than net losses.

B. net income is less than dividends.

C. net income is greater than dividends.

D. a net loss is less than dividends.

Score: 10/10

7.

 

An income statement shows

Student Response

A. revenues, expenses, and net income.

B. revenues, liabilities, and stockholders' equity.

C. expenses, dividends, and stockholders' equity.

D. assets, liabilities, and stockholders' equity.

Score: 10/10

8.

 

If assets increase, then which of the following could have happened?

Student Response

11

Page 12: Accounting and Finance

A. stockholders' equity decrease

B. expenses increase

C. liabilities increase

D. liabilities decrease

Score: 0/10

9.

 

If total liabilities increased by $25,000 and stockholders' equity increased by $5,000 during a period of time, then total assets must change by what amount and direction during that same period?

Student Response

A. $20,000 decrease

B. $20,000 increase

C. $30,000 increase

D. $25,000 increase

Score: 0/10

10.

  If total liabilities decreased by $15,000 and stockholders' equity decreased by $5,000 during a period of time, then total assets must change by what amount and direction during that same period?

12

Page 13: Accounting and Finance

Student Response

A. $10,000 decrease

B. $20,000 decrease

C. $10,000 decrease

D. $20,000 increase

Score: 0/10

 

13

Page 14: Accounting and Finance

Title: > Quiz 2

Started: August 23, 2012 12:21 PM

Submitted: August 23, 2012 1:21 PM

Time spent: 01:00:04

Total score: 60/100 = 60% Total score adjusted by 0.0 Maximum possible score: 100

1.

 

A current asset is

Student Response

A. expected to be converted to cash or used in the business within a relatively short period of time.

B. an asset which is currently being used to produce a product or service.

C. usually found as a separate classification in the income statement.

D. the last asset purchased by a business.

Score: 10/10

2.

  Accounts payable would be classified as a

Student Response

A. current asset.

B. current liability.

14

Page 15: Accounting and Finance

C. plant, property, & equipment.

D. long-term liability.

Score: 10/10

3.

  Able Auto Supplies has the following accounts and their balances:Accounts Payable $65,000Accounts Receivable $60,000Accumulated Depreciation $20,000Building $100,000Cash $50,000Common Stock $120,000Inventory $70,000Land $95,000Land held for future use $80,000Mortgage Payable $90,000Prepaid Insurance $30,000Retained Earnings $250,000Salaries Payable $10,000Trademark $70,000

The total dollar amount of assets to be classified as current assets is

Student Response

A. $210,000.

B. $290,000.

C. $140,000.

D. $150,000.

Score: 10/10

15

Page 16: Accounting and Finance

4.

 

Acme Auto Supplies listed the following accounts on their December 31, 2007

Cash $60,000Prepaid Insurance $40,000Accounts Receivable $50,000Inventory $70,000Land held for investment $80,000Land $95,000Building $100,000Less: Accumulated Depreciation <$30,000>Trademark $70,000Accounts Payable $65,000Salaries Payable $10,000Mortgage Payable $90,000Common Stock $120,000Retained Earnings $250,000

The total dollar amount of assets to be classified as property, plant, and equipment is

Student Response

A. $195,000.

B. $165.000.

C. $315,000.

D. $245,000.

Score: 0/10

16

User, 08/23/12,
#4 PP&E=Land 95k, Building 100k, Accumulated Depreciation (contra PP&E asset) <30K>=$165000
Page 17: Accounting and Finance

5.

 

Acme Auto Supplies listed the following accounts on their December 31, 2007

Cash $60,000Prepaid Insurance $40,000Accounts Receivable $50,000Inventory $70,000Land held for investment $80,000Land $95,000Building $100,000Less: Accumulated Depreciation <$30,000>Trademark $70,000Accounts Payable $65,000Salaries Payable $10,000Mortgage Payable $90,000Common Stock $120,000Retained Earnings $250,000

The total dollar amount of asssets to be classified as investments is

Student Response

A. $0.

B. $180,000.

C. $150,000.

D. $80,000.

Score: 0/10

6.

  Acme Auto Supplies listed the following accounts on their December 31, 2007

Cash $60,000

17

User, 08/23/12,
#5 WC=CA-CLCurrent Assets: Cash 60k, Prepaid Insurance 40k, Accounts Receivable 50k, Inventory 70k=$220000Current Liabilities: Accounts Payable 65k, Salaries Payable 10k=$75000WC=220000-75000=$145000
Page 18: Accounting and Finance

Prepaid Insurance $40,000Accounts Receivable $50,000Inventory $70,000Land held for investment $80,000Land $95,000Building $100,000Less: Accumulated Depreciation <$30,000>Trademark $70,000Accounts Payable $65,000Salaries Payable $10,000Mortgage Payable $90,000Common Stock $120,000Retained Earnings $250,000

The total amount of working capital is

Student Response

A. $150,000.

B. $155,000.

C. $60,000.

D. $145,000.

Score: 0/10

7.

  Acme Auto Supplies listed the following accounts on their December 31, 2007

Cash $60,000Prepaid Insurance $40,000Accounts Receivable $50,000Inventory $70,000Land held for investment $80,000Land $95,000

18

Page 19: Accounting and Finance

Building $100,000Less: Accumulated Depreciation <$30,000> $70,000Accounts Payable $65,000Salaries Payable $10,000Mortgage Payable $90,000Common Stock $120,000Retained Earnings $250,000

The current ratio is

Student Response

A. 1.86:1.

B. 3.38:1.

C. 2.93:1.

D. 2.00:1.

Score: 0/10

8.

  Based on the following data, what is the debt to total assets ratio (to the nearest percent)100%?<br>Accounts Payable $31,000<br>Accounts Receivable $57,000<br>Cash $15,000<br>Intangible Assets $50,000<br>Inventory $69,000<br>Long-Term Investments $80,000<br>Long-Term Liabilities $100,000<br>Short-Term Investments $40,000<br>Notes Payable $28,000<br>Plant Assets $670,000<br>

19

Page 20: Accounting and Finance

Prepaid Expenses $1,000

Student Response

A. 51%

B. 54%

C. 16%

D. 100%

Score: 10/10

9.

 

Current assets $7,000Current liabilities $4,000Average assets $40,000Total assets $30,000Net income $12,000Stockholders' equity $27,000Total liabilities $9,000

Average common shares outstanding was 10,000

What is the debt to total assets?

Student Response

A. 30%

B. 75%

C. 22.5%

20

Page 21: Accounting and Finance

D. 13%

Score: 10/10

10.

 

An example of a liquidity ratio would be

Student Response

A. earnings per share.

B. current ratio.

C. debt to total assets ratio.

D. times interest earned ratio.

Score: 10/10

 

Chapter 3

AP3-5A

Towne Architects incorporated as licensed architects on April 1, 2012. During the first month of the operation of the business, these events and transactions occurred:

April 1   Stockholders invested $20,283 cash in exchange for common stock of the corporation.1   Hired a secretary-receptionist at a salary of $507 per week, payable monthly.2   Paid office rent for the month $1,217.3   Purchased architectural supplies on account from Spring Green Company $1,352.

10   Completed blueprints on a carport and billed client $2,028 for services.

21

Page 22: Accounting and Finance

11   Received $676 cash advance from J. Madison to design a new home.20   Received $3,110 cash for services completed and delivered to M. Svetlana.30   Paid secretary-receptionist for the month $2,028.30   Paid $406 to Spring Green Company for accounts payable due.

Your answer is correct.

   Journalize the transactions. (If no entry is required type No entry for the account and 0 for the amount.)Date Account/Description Debit Credit

April 1  

           

 

April 1  

           

 

April 2  

           

 

April 3  

           

 

April 10  

           

 

April 11  

           

 

April 20  

           

 

April 30  

           

 

April 30  

           

 

Your answer is correct.

   Complete the following trial balance. (Hint: Post the above transactions to the ledger T-accounts and

22

Cash 20283

Common stock 20283

No entry 0

No entry 0

Rent expense 1217

Cash 1217

Supplies 1352

Accounts payable 1352

Accounts receivable 2028

Service revenue 2028

Cash 676

Unearned Revenue 676

Cash 3110

Service revenue 3110

Salaries expense 2028

Cash 2028

Accounts payable 406

Cash 406

Page 23: Accounting and Finance

use the balances to complete the trial balance.) (If answer is zero please enter 0, do not leave any fields blank.)

TOWNE ARCHITECTS INC.Trial BalanceApril 30, 2012

Debit Credit

Cash$ $

Accounts receivable

Supplies

Accounts payable

Unearned Revenue

Common stock

Service revenue

Salaries Expense

Rent expense

$ $

Click here if you would like to Show Work for this question

Question Attempts: Unlimited  

23

20418 0

2028 0

1352 0

0 946

0 676

0 20283

0 5138

2028 0

1217 0

27043 27043

User, 08/23/12,
Make sure you pay attention to wording! “Purchased” supplies from Spring Green Company, but ONLY paid back smaller amount for accounts payable due!
User, 08/23/12,
Complete credit side of Trail Balance to get total credit. Then, add all debits to subtract from total credit to get CASH AMOUNT!
Page 24: Accounting and Finance

Copyright © 2000-2012 by John Wiley & Sons, Inc. or related companies. All rights reserved.

Jump to Navigation Frame Jump to Content Frame

   Your location: Learning Modules › Chapter 3 › Quiz 3 › >Quiz 3 › Assessments › View All Submissions › View Attempt

View Attempt 1 of 1

Title: >Quiz 3

Started: August 28, 2012 10:56 AM

Submitted: August 28, 2012 11:40 AM

Time spent: 00:43:41

Total score: 80/100 = 80% Total score adjusted by 0.0 Maximum possible score: 100

1.

  The purchase of an asset on credit

Student Response

A. increases assets and liabilities.

B. decreases assets and increases liabilities.

C. increases assets and stockholders' equity.

D. leaves total assets unchanged.

24

Page 25: Accounting and Finance

Score: 10/10

2.

 

When collection is made on Accounts Receivable

Student Response

A. stockholders' equity will increase.

B. total assets will remain the same.

C. total assets will increase.

D. total assets will decrease.

Score: 0/10

3.

 

A revenue generally

Student Response

A. increase assets and liabilities.

B. increases assets and decreases stockholders' equity.

C. increases assets and stockholders' equity.

D. leaves total assets unchanged.

25

Page 26: Accounting and Finance

Score: 10/10

4.

 

A payment of a portion of accounts payable will

Student Response

A. increase liabilities.

B. decrease net income.

C. not affect stockholders' equity.

D. not affect total assets.

Score: 10/10

5.

  Which of the following describes the classification and normal balance of the retained earnings account?

Student Response

A. Expense, debit

B. Revenue, credit

C. Stockholders' Equity, credit

D. Asset, debit

Score: 10/10

26

Page 27: Accounting and Finance

6.

 

Which of the following describes the classification and normal balance of the unearned revenue account?

Student Response

A. Expense, debit

B. Revenue, credit

C. Asset, debit

D. Liability, credit

Score: 10/10

7.

 

Which accounts normally have debit balances?

Student Response

A. Assets, expenses, and dividends.

B. Assets, expenses, and retained earnings.

C. Assets, liabilities, and dividends.

D. Assets, expenses, and revenues.

Score: 10/10

27

Page 28: Accounting and Finance

8.

 

Which of the following correctly identifies normal balances of accounts?

Student Response

A. Assets, debit; Liabilities, credit; Common Stock, credit; Revenues, credit; Expenses, credit

B. Assets, debit; Liabilities, credit; Common Stock, credit; Revenues, debit; Expenses, credit

C. Assets, credit; Liabilities, debit; Common Stock, debit; Revenues, credit; Expenses, debit

D. Assets, debit; Liabilities, credit; Common Stock, credit; Revenues, credit; Expenses, de

Score: 0/10

9.

  When a company performs a service but has not yet received payment, it

Student Response

A. makes no entry until cash is received.

B. debits service revenues and credits accounts receivable.

C. debits service revenues and credits accounts payable.

D. debits accounts receivable and credits service revenue.

Score: 10/10

28

Page 29: Accounting and Finance

10.

 

Which account below is not a subdivision of stockholders' equity?

Student Response

A. Liabilities

B. Expenses

C. Revenues

D. Dividends

Score: 10/10

Chapter 4

AP4-8A

Dana La Fontsee opened Pro Window Washing Inc. on July 1, 2012. During July the following transactions were completed.

 

July 1 Issued 11,979 shares of common stock for $11,979 cash.July 1 Purchased used truck for $9,979, paying $2,046 cash and the balance on account.July 3 Purchased cleaning supplies for $908 on account.July 5 Paid $1,881 cash on 1-year insurance policy effective July 1.July 12 Billed customers $3,281 for cleaning services.July 18 Paid $1,263 cash on amount owed on truck and $539 on amount owed on cleaning

supplies.July 20 Paid $2,139 cash for employee salaries.July 21 Collected $1,556 cash from customers billed on July 12.July 25 Billed customers $2,631 for cleaning services. July 31 Paid $377 for gas and oil used in the truck during month.July 31 Declared and paid a $634 cash dividend.

29

Page 30: Accounting and Finance

Your answer is correct.

   Journalize the July transactions. (List multiple debit/credit entries from largest to smallest amount, e.g. 10, 5, 2.)

Date Account/Description Debit Credit

July 1  

           

 

  (To record the issuance of stock)    

July 1  

           

 

            

 

  (To record the purchase of a truck)    

July 3  

          

 

July 5  

          

 

July 12  

           

 

July 18  

           

 

July 20  

           

 

July 21  

           

 

July 25  

           

 

July 31  

           

 

  (Paid for gas and oil)    

July 31  

           

 

  (Declared and paid a dividend)    

30

Cash 11979

Common Stock 11979

Equipment 9979

Accounts Payable 7933

Cash 2046

Cleaning Supplies 908

Accounts Payable 908

Prepaid Insurance 1881

Cash 1881

Accounts Receivable 3281

Service Revenue 3281

Accounts Payable 1802

Cash 1802

Salaries Expense 2139

Cash 2139

Cash 1556

Accounts Receivable 1556

Accounts Receivable 2631

Service Revenue 2631

Gas & Oil Expense 377

Cash 377

Dividends 634

Cash 634

Page 31: Accounting and Finance

Your answer is correct.

   Journalize the following adjustments. (Round answers to 0 decimal places, e.g. 2,520.)

1. Services provided but unbilled and uncollected at July 31 were $1,805.2. Depreciation on equipment for the month was $254.3. One-twelfth of the insurance expired. 4. An inventory count shows $360 of cleaning supplies on hand at July 31.5. Accrued but unpaid employee salaries were $406.Date Account/Description Debit Credit

1. July 31  

            

 

2. July 31  

           

 

3. July 31  

           

 

4. July 31  

           

 

5. July 31  

           

 

Your answer is correct.

   Post the July transactions to the ledger accounts. (Use T accounts.) Post adjusting entries to the T accounts. Post closing entries and complete the closing process.(If answer is zero, please enter 0, do not leave any fields blank.)

Cash   Accounts Payable

 7/1   7/1     7/18   7/1  

 7/21   7/5        7/3  

    7/18        7/31 Bal.  

31

Accounts Receivable 1805.

Service Revenue 1805.

Depreciation Expense 254.

Accumulated Depreciation 254.

Insurance Expense 156.

Prepaid Insurance 156.

Cleaning Supplies Expense 548.

Cleaning Supplies 548.

Salaries Expense 406.

Salaries Payable 406.

11979 2046 1802 7933

1556 1881

908

1802 7039

Page 32: Accounting and Finance

    7/20          

    7/31    Salaries Payable

    7/31        7/31  

 7/31 Bal.          

 7/31 Bal.  

               Accounts Receivable   Common Stock

 7/12   7/21        7/1  

 7/25           7/31 Bal.  

 7/31              7/31 Bal.       Retained Earnings

         7/31   7/31  

Cleaning Supplies       7/31 Bal.  

 7/3   7/31           7/31 Bal.      

Dividends

         7/31   7/31  

    7/31 Bal      

Prepaid Insurance    

 7/5   7/31    Income Summary

 7/31 Bal.   

MUST BE FIGURED OUT

BY DOING THE INCOME

STATEMENT BELOW 1ST!    7/31   7/31  

         7/31     

Equipment       7/31 Bal.  

 7/1              7/31 Bal.      

Service Revenue

         7/31   7/12  

Accumulated Depreciation - Equipment       7/25  

32

2139

377

634 406

4656

406

3281 1556

11979

2631

11979

1805

6161

634

3837

3203

908

548

360

634 634

0

1881

156

1725 1) 3880 7717

2) 3837

0

9979

9979

7717 3281

2631

User, 08/27/12,
Taken from both the Journalize Adjustments and Journalize Transactions
User, 08/27/12,
Net Income
User, 08/27/12,
Service Revenue
User, 08/27/12,
Total Expense
Page 33: Accounting and Finance

    7/31        7/31  

    7/31 Bal.       

 7/31 Bal.  

              Gas and Oil Expense   Insurance Expense

 7/31   7/31     7/31   7/31   7/31 Bal.      

 7/31 Bal.     

              Cleaning Supplies Expense   Salaries Expense

 7/31   7/31     7/20   7/31   7/31 Bal.        7/31     

        7/31 Bal.     

Depreciation Expense         

 7/31   7/31       

    7/31 Bal.     

         

Your answer is correct.

   Complete the Trial Balance and Adjusted Trial Balance at July 31. (If an answer is zero, please enter 0, do not leave any fields blank.)

PRO WINDOW WASHING INC.Trial BalanceJuly 31, 2012

  Before Adjustment After Adjustment  Debit   Credit Debit   Credit

Cash$ $ $ $

Accounts Receivable

Cleaning Supplies

Prepaid Insurance

Equipment

Acc. Depr. - Equipment

Accounts Payable

33

254 1805

254 0

377

377

156

156

0 0

548

548

2139 2545

0 406

0

254

254

0

4656 0 4656 0

4356 0 6161 0

908 0 360 0

1881 0 1725 0

9979 0 9979 0

0 0 0 254

0 7039 0 7039

Page 34: Accounting and Finance

Salaries Payable

Common Stock

Dividends

Service Revenue

Salaries Expense

Gas & Oil Expense

Depreciation Expense

Insurance Expense

Cleaning Supplies Expense

 $ $ $ $

Your answer is correct.

   Complete the income statement and a retained earnings statement for July and a classified balance sheet at July 31. (List expenses from largest to smallest amount, e.g. 10, 5, 2. List assets in order of liquidity and liabilities from largest to smallest amount, e.g. 10, 5, 2.)

PRO WINDOW WASHING INC.Income Statement

Revenues

       $Expenses

       $

      

      

      

      

            Total Expenses

34

0 0 0 406

0 11979 0 11979

634 0 634 0

0 5912 0 7717

2139 0 2545 0

377 0 377 0

0 0 254 0

0 0 156 0

0 0 548 0

24930 24930 27395 27395

For the Month Ended July 31, 2012

Service revenue 7717

Salaries Expense 2545

Cleaning Supplies Expense 548

Gas & Oil Expense 377

Depreciation Expense 254

Insurance Expense156

3880

User, 08/27/12,
Credits vs. Debits
Page 35: Accounting and Finance

Net Income$

PRO WINDOW WASHING INC.Retained Earnings Statement

$

Add: 

Less: 

$PRO WINDOW WASHING INC.

Balance Sheet

AssetsCurrent assets

       $

      

      

      

            Total current assets$

Property, plant, and equipment

      

   Less: 

            Total assets$

Liabilities and Stockholders’ EquityCurrent liabilities

       $

      

           Total current liabilities$

Stockholders’ equity

35

3837

For the Month Ended July 31, 2012

Retained earnings, July 1 0

Net income3837

3837

Dividends634

Retained earnings, July 313203

July 31, 2012

Cash 4656

Accounts Receivable 6161

Cleaning supplies 360

Prepaid Insurance1725

12902

Equipment 9979

Accumulated Depreciation254 9725

22627

Accounts Payable 7039

Salaries Payable406

7445

Page 36: Accounting and Finance

      

      

           Total stockholders’ equity

            Total liabilities and stockholders’ equity$

Your answer is correct.

   Journalize the post closing entries. (List multiple debit/credit entries from largest to smallest amount, e.g. 10, 5, 2.)

Date Account/Description Debit Credit

July 31  

           

 

  (To close revenue accounts.)    

July 31  

           

 

           

 

           

 

           

 

           

 

  (To close expense accounts.)    

July 31  

           

 

  (To close income summary. )    

July 31  

           

 

  (To close dividends.)    

36

Common stock 11979

Retained earnings3203

15182

22627

Service Revenue 7717

Income Summary 7717

Income Summary 1) 3880

Salaries Expense 2545

Cleaning Supplies Expense 548

Gas & Oil Expense 377

Depreciation Expense 254

Insurance Expense 156

Income Summary 2) 3837

Retained Earnings 3837

Retained Earnings 634

Dividends 634

Page 37: Accounting and Finance

Your answer is correct.

   Complete the post closing Trial Balance below. (If answer is zero please enter 0, do not leave any fields blank.)

PRO WINDOW WASHING INC.Post-Closing Trial Balance

July 31, 2012Debit Credit

Cash$ $

Accounts Receivable

Cleaning Supplies

Prepaid Insurance

Equipment

Accumulated Depreciation

Accounts Payable

Salaries Payable

Common Stock

Retained Earnings

Dividends

Service Revenue

Salaries Expense

Gas & Oil Expense

Depreciation Expense

Insurance Expense

Cleaning Supplies Expense

$ $

Click here if you would like to Show Work for this question

37

4656 0

6161 0

360 0

1725 0

9979 0

0 254

0 7039

0 406

0 11979

03203

0 0

0 0

0 0

0 0

0 0

0 0

0 0

22881 22881

User, 08/27/12,
All entries are recorded from the Balance in Ledger Accounts.
Page 38: Accounting and Finance

Question Attempts: Unlimited  

Copyright © 2000-2012 by John Wiley & Sons, Inc. or related companies. All rights reserved.

Title: > Quiz 4

Started: August 28, 2012 11:51 AM

Submitted: August 28, 2012 12:23 PM

Time spent: 00:32:18

Total score: 90/100 = 90% Total score adjusted by 0.0 Maximum possible score: 100

1.

  In a service-type business, revenue is considered earned

Student Response

A. at the end of the year.

B. when cash is received.

38

Page 39: Accounting and Finance

C. at the end of the month.

D. when the service is performed.

Score: 10/10

2.

 

Javier's Tune-Up Shop follows the revenue recognition principle. Javier services a car on August 31. The customer picks up the vehicle on September 1 and mails the payment to Javier on September 5. Javier receives the check in the mail on September 6. When should Javier show that the revenue was earned?

Student Response

A. September 6

B. September 1

C. September 5

D. August 31

Score: 10/10

3.

 

A flower shop makes a large sale for $1,000 on November 30. The customer is sent a statement on December 5 and a check is received on December 10. The flower shop follows GAAP and applies the revenue recognition principle. When is the $1,000 considered to be earned?

39

Page 40: Accounting and Finance

Student Response

A. November 30

B. December 5

C. December 1

D. December 10

Score: 10/10

4.

 

Reese Company purchased office supplies costing $4,000 and debited Office Supplies for the full amount. At the end of the accounting period, a physical count of office supplies revealed $1,600 still on hand. The appropriate adjusting journal entry to be made at the end of the period would be

Student Response

A. Debit Office Supplies Expense, $2,400; Credit Office Supplies, $2,400.

B. Debit Office Supplies Expense, $1,600; Credit Office Supplies, $1,600.

C. Debit Office Supplies, $2,400; Credit Office Supplies Expense, $2,400.

D. Debit Office Supplies, $1,600; Credit Office Supplies Expense, $1,600.

Score: 10/10

5.

  Which of the following acounts would not likely need to be adjusted at year end?

40

Page 41: Accounting and Finance

Student Response

A. Office Supplies

B. Prepaid Advertising

C. Unearned Revenue

D. Land

Score: 10/10

6.

 

An adjusting entry can include a

Student Response

A. debit to an asset and a credit to a revenue.

B. debit to a revenue and a credit to an asset.

C. credit to a liability and a debit to a revenue.

D. debit to an expense and a credit to a revenue.

Score: 0/10

7.

  Accrued revenues are

41

Page 42: Accounting and Finance

Student Response

A. earned and recorded as liabilities before they are received.

B. earned and already received and recorded.

C. received and recorded as liabilities before they are earned.

D. earned but not yet received or recorded.

Score: 10/10

8.

 

The Village Laundry Company purchased $6,500 worth of laundry supplies on June 2 and recorded the purchase as an asset. On June 30, an inventory of the laundry supplies indicated only $3,000 on hand. The adjusting entry that should be made by the company on June 30 is

Student Response

A. debit Laundry Supplies Expense, $3,500; credit Laundry Supplies, $3,000

B. debit Laundry Supplies, $3,500; credit Laundry Supplies Expense, $3,500

C. debit Laundry Supplies Expense, $3,500; credit Laundry Supplies, $3,500

D. debit Laundry Supplies Expense, $3,000; credit Laundry Supplies, $3,000

Score: 10/10

9.

  Accrued expenses are

42

Page 43: Accounting and Finance

Student Response

A. incurred and already paid or recorded.

B. paid and recorded in an asset account before they are used or consumed.

C. paid and recorded in an asset account after they are used or consumed.

D. incurred but not yet paid or recorded.

Score: 10/10

10.

 

The balance in the prepaid rent account before adjustment at the end of the year is $9,000 and represents three months rent paid on December 1. The adjusting entry required on December 31 is

Student Response

A. debit Rent Expense, $9,000; credit Prepaid Rent $9,000

B. debit Prepaid Rent, $6,000, credit Rent Expense, $6,000

C. debit Prepaid Rent, $3,000; credit Rent Expense $3,000

D. debit Rent Expense, $3,000; credit Prepaid Rent $3,000

Score: 10/10

43

Page 44: Accounting and Finance

Sample Test 1

ABE1-9

Your answer is correct.

   

At the beginning of the year, Peale Company had total assets of $900,476 and total liabilities of $534,524.

(a) If total assets increased $165,721 during the year and total liabilities decreased $85,034, what is the amount of stockholders' equity at the end of the year?

(b) During the year, total liabilities increased $98,461 and stockholders' equity decreased $64,376.  What is the amount of total assets at the end of the year.

(c) If total assets decreased $81,271 and stockholders' equity increased $110,466 during  the year, what is the amount of total liabilities at the end of the year?

Assets = Liabilities + Stockholders' Equity

(a)$

 $

 $

(b)$

 $

 $

(c)$

 $

 $

Click here if you would like to Show Work for this question

AE1-8

Your answer is correct.

   

The items and amounts below were taken from Linus Inc.'s 2012 income statement and balance sheet.

(a)In each, case, identify whether the item is an asset, liability, stockholder's equity, revenue, or expense item.

Cash and short-term investments $89,064

Retained earnings 127,406

Cost of goods sold 445,816

Selling, general, and administrative expenses 124,171

44

1066197 449490 616707

934561 632985 301576

819205 342787 476418

Asset

Stockholders' equity

Expense

Expense

 

User, 08/29/12,
Must subtract initial total asset from the initial total liabilities to get the initial Stockholder’s Equity.
Page 45: Accounting and Finance

Prepaid expenses 8,612

Inventories 74,381

Receivables 97,431

Sales revenue 588,225

Income taxes payable 6,818

Accounts payable 49,962

Franchising revenues 5,682

Interest expense 2,651

(b)Prepare an income statement for Linus Inc. for the year ended December 31, 2012. (List multiple entries from largest to smallest amounts, e.g. 10, 5, 1.)

LINUS INC.Income Statement

Revenues:

       $ 

       

          Total Revenue$

Expenses:

      

      

      

          Total expenses

Net income$

Click here if you would like to Show Work for this question

AE2-5

Your answer is correct.

   

These items are taken from the financial statements of Victory Co. at December 31, 2012.

Building $87,179Accounts receivable 14,944Prepaid insurance 5,560Cash 16,621Equipment 96,655

45

Asset

Asset

Asset

Revenue

Liability

Liability

Revenue

Expense

For the Year Ended December 31, 2012

Sales revenue 588225

Franchising revenues5682

593907

Cost of goods sold 445816

Selling, general and administrative expenses 124171

Interest expense2651

572638

21269

 

 

Page 46: Accounting and Finance

Land 59,180Insurance expense 780Depreciation expense 4,420Interest expense 2,597Common stock 72,726Retained earnings (January 1, 2012) 45,092Accumulated depreciation—building 38,030Accounts payable 11,267Note payable 79,071Accumulated depreciation—equipment 18,701Interest payable 4,118Bowling revenues 18,931Assume that $13,151 of the note payable will be paid in 2013.  Complete the following classified balance sheet. (List current assets in order of liquidity and liabilities from largest to smallest amounts, e.g. 10, 5, 1.)

VICTORY COMPANY

Balance Sheet

Assets

Current assets

     $

    

    

      Total current assets$

Property, plant, and equipment

    

    $

    Less: 

    

    Less: 

       Total assets$

 

Liabilities and Stockholders' Equity

Current liabilities

     $

    

46

December 31, 2012

Cash 16621

Accounts receivable 14944

Prepaid insurance5560

37125

Land 59180

Building 87179

Accumulated depreciation-building38030

49149

Equipment 96,655

Accumulated depreciation-equipment18,701 77954 186283

223408

Current portion of note payable 13151

Accounts payable 11,267

User, 08/29/12,
Subtract total asset from total liabilities to get total stockholder’s equity.
Page 47: Accounting and Finance

    

       Total current liabilities$

Long-term liabilities

    

      Total liabilities

Stockholders' equity

    

      

       Total stockholders' equity

       Total liabilities and stockholders' equity$

Click here if you would like to Show Work for this question

AE3-3

Your answer is correct.

   

During 2012, its first year of operations as a delivery service, Underwood Corp. entered into the following transactions.

1. Issued shares of common stock to investors in exchange for $103,869 in cash.2. Borrowed $47,639 by issuing bonds.3. Purchased delivery trucks for $64,169 cash.4. Received $16,131 from customers for services provided.5. Purchased supplies for $5,123 on account.6. Paid rent of $6,424.7. Performed services on account for $13,931.8. Paid salaries of $30,984.9. Paid a dividend of $13,568 to shareholders.

Using the following tabular analysis, show the effect of each transaction on the accounting equation. (If answer is zero, please enter 0, do not leave any fields blank. For negative numbers use either a negative sign preceding the number, e.g. -45 or parenthesis, e.g. (45).)

  Assets   Liabilities   Stockholders' Equity

47

Interest payable4,118

28536

Note payable65920

94456

Common stock 72,726

Retained earnings56226

128952

223408

User, 08/29/12,
Total= Note payable – portion of not payable.
Page 48: Accounting and Finance

Cash +Accounts

Receivable + Supplies +

Property, Plant &

Equipment =Accounts Payable +

Bonds Payable +

Common Stock +

Retained Earnings

1.              

2.              

3.              

4.              

5.              

6.              

7.              

8.              

9.              

Totals

             

AE3-8

Your answer is correct.

   

This information relates to Plunkett Real Estate Agency.

Oct. 1 Stockholders invested $30,592 in exchange for common stock of the corporation.Oct. 2 Hires an administrative assistant at an annual salary of $44,690.Oct. 3 Buys office furniture for $4,961, on account.Oct. 6 Sells a house and lot for M.E. Petty; commissions due from Petty, $14,759 (not paid by Petty at

this time).Oct. 10 Receives cash of $197 as commission for acting as rental agent renting an apartment.Oct. 27 Pays $800 on account for the office furniture purchased on October 3.Oct. 30 Pays the administrative assistant $3,518 in salary for October.

Journalize the transactions. (If there is no entry for the transaction enter No entry as the account and 0 for the amount.)

Date Account/Description Debit Credit

Oct. 1  

           

 

Oct. 2  

           

 

Oct. 3   

48

103869 0 0 0 0 0 103869 0

47639 0 0 0 0 47639 0 0

-64169 0 0 64169 0 0 0 0

16131 0 0 0 0 0 0 16131

0 0 5123 0 5123 0 0 0

-6424 0 0 0 0 0 0 -6424

0 13,931 0 0 0 0 0 13931

-30984 0 0 0 0 0 0 -30984

-13568 0 0 0 0 0 0 -13568

52494 13931 5123 64169 5123 47639 103869 -20914

Cash 30592

Common stock 30592

No entry 0

No entry 0

Office furniture 4961

Page 49: Accounting and Finance

           

 

Oct. 6  

           

 

Oct. 10  

           

 

Oct. 27  

           

 

Oct. 30  

           

 

Click here if you would like to Show Work for this question

AE4-9

Your answer is correct.

   The ledger of Sagovic Rental Agency on March 31 of the current year includes these selected accounts before adjusting entries have been prepared.

  Debits CreditsPrepaid Insurance $3,600  Supplies 3,660  Equipment 25,000  Accumulated Depreciation-Equipment   $8,400Notes Payable   20,000Unearned Rent Revenue   11,824Rent Revenue   60,000Interest Expense 0  Wage Expense 14,000  An analysis of the accounts shows the following.

1. The equipment depreciates $219 per month.2. Half of the unearned rent revenue was earned during the quarter.3. Interest of $575 is accrued on the notes payable.4. Supplies on hand total $843.5. Insurance expires at the rate of $229 per month.

Prepare the adjusting entries at March 31, assuming that adjusting entries are made quarterly.

49

Accounts payable 4961

Accounts receivable 14759

Service revenue 14759

Cash 197

Service revenue 197

Accounts payable 800

Cash 800

Salaries expense 3518

Cash 3518

 

Page 50: Accounting and Finance

Date Account/Description Debit Credit1) Mar. 31

 

           

 

2) Mar. 31

 

           

 

3) Mar. 31

 

           

 

4) Mar. 31

 

           

 

5) Mar. 31

 

           

 

AE4-10

Your answer is correct.

   

Adam Singh, D.D.S., opened an incorporated dental practice on January 1, 2012. During the first month of operations the following transactions occurred:

1. Performed services for patients who had dental plan insurance. At January 31, $716 of such services was earned but not yet billed to the insurance companies.

2. Utility expenses incurred but not paid prior to January 31 totaled $645.3. Purchased dental equipment on January 1 for $83,870, paying $20,180 in cash and signing a

$63,690, 3-year note payable (Interest is paid each December 31). The equipment depreciates $562 per month. Interest is $681 per month.

4. Purchased a 1-year malpractice insurance policy on January 1 for $28,776.5. Purchased $2,070 of dental supplies (recorded as increase to Supplies). On January 31 determined

that $632 of supplies were on hand.

Prepare the adjusting entries on January 31.

Date Account/Description Debit Credit1) Jan. 31

 

           

 

2) Jan. 31

 

50

Depreciation Expense 657

Accumulated Depreciation-Equipment 657

Unearned Rent Revenue 5912

Rent Revenue 5912

Interest Expense 575

Interest Payable 575

Supplies Expense 2817

Supplies 2817

Insurance Expense 687

Prepaid Insurance 687

Accounts Receivable 716

Service Revenue 716

Utilities Expense 645

User, 09/01/12,
Multiply by 3 = Quarter Total
User, 09/01/12,
Supplies Initial Amount – Supplies Total Left on Hand.
User, 09/01/12,
Multiply by 3 = Quarter Total
Page 51: Accounting and Finance

           

 

3) Jan. 31

 

           

 

  (To record depreciation.)    

  

 

           

 

  (To record interest.)    4) Jan. 31

 

           

 

5) Jan. 31

 

           

 

Click here if you would like to Show Work for this question

Chapter 5

AP5-2A

Your answer is correct.

   

Hayes Warehouse distributes hardback books to retail stores and extends credit terms of 2/10, n/30 to all of its customers. During the month of June the following merchandising transactions occurred.

June 1 Purchased books on account for $960 (including freight) from Brooks Publishers, terms 3/10, n/30.

3 Sold books on account to the Mission Viejo Bookstore for $1,725. The cost of the merchandise sold was $690.

6 Received $78 credit for books returned to Brooks Publishers.9 Paid Brooks Publishers in full.

15 Received payment in full from the Mission Viejo Bookstore.17 Sold books on account to Book Nook Bookstore for $2,100. The cost of the merchandise sold was

$840.20 Purchased books on account for $720 from Cook Book Publishers, terms 3/15, n/30.24 Received payment in full from Book Nook Bookstore.26 Paid Cook Book Publishers in full.28 Sold books on account to NewTown Bookstore for $1,905. The cost of the merchandise sold was

$762.30 Granted NewTown Bookstore $195 credit for 13 books returned costing $78.

51

Utilities Payable 645

Depreciation Expense 562

Accumulated Depreciation-Dental Equip. 562

Interest Expense 681

Interest Payable 681

Insurance Expense 2398

Prepaid Insurance 2398

Supplies Expense 1438

Supplies 1438

User, 09/01/12,
Divided Total insurance on January 1st by 12 for months to get amount per month paid for adjustment.
Page 52: Accounting and Finance

Journalize the transactions for the month of June for Hayes Warehouse, using a perpetual inventory system. (List multiple debit/credit entries from largest to smallest amount, e.g. 10, 5, 2. Round all answers to 0 decimal places, ie. 2,555.)

Date Account/Description Debit Credit

June 1  

          

 

June 3  

           

 

  (To record the sale.)    

   

          

 

  (To record the cost of inventory.)    

June 6  

         

 

June 9  

           

 

           

 

June 15  

          

 

June 17  

          

 

  (To record the sale.)    

   

          

 

  (To record the cost of inventory.)    

June 20  

          

 

June 24  

   

          

 

June 26  

          

 

          

 

June 28  

52

Merchandise Inventory 960

Accounts Payable 960

Accounts Receivable 1725

Sales 1725

Cost of Goods Sold 690

Merchandise Inventory 690

Accounts Payable 78

Merchandise Inventory 78

Accounts Payable 882

Cash 855.54

Merchandise Inventory 26.46

Cash 1725

Accounts Receivable 1725

Accounts Receivable 2100

Sales 2100

Cost of Goods Sold 840

Merchandise Inventory 840

Merchandise Inventory 720

Accounts Payable 720

Cash 2058

Sales Discounts 42

Accounts Receivable 2100

Accounts Payable 720

Cash 698.40

Merchandise Inventory 21.60

Accounts Receivable 1905

User, 09/01/12,
720 x (3/10) Original Terms because it’s after 10 day period.
User, 09/01/12,
720 – 21.60 = Cash Amount
User, 09/01/12,
2100 x (2/10) @ Top of June 1 terms = 42. MUST MOVE DECIMAL 1 PLACE TO THE LEFT!
User, 09/01/12,
2100 – 42 = 2058
User, 09/01/12,
882 x (3/10) terms for June 1 = 26.46. MAKE SURE TO MOVE DECIMAL OVER 2 PLACES TO THE LEFT!
User, 09/01/12,
882 (Accounts Payable) – 26.46 (Merchandise Inventory) = 855.54
User, 09/01/12,
960 (Purchased Books on Accounts) – 78 (Received Credits for Books) = 882
Page 53: Accounting and Finance

           

 

  (To record the sale.)    

   

          

 

  (To record the cost of inventory.)    

June 30  

           

 

  (To record the credit for returned merchandise.)    

   

          

 

  (To record the merchandise returned.)    

Click here if you would like to Show Work for this question

AP5-9A

At the beginning of the current season on April 1, the ledger of Thousand Oaks Pro Shop showed Cash $3,057; Merchandise Inventory $4,057; and Common Stock $7,114. The following transactions were completed during April 2012.

Apr. 5 Purchased golf bags, clubs, and balls on account from Ryder Co. $1,942, terms 3/10, n/60.7 Paid freight on Ryder purchase $116.9 Received credit from Ryder Co. for merchandise returned $270.

10 Sold merchandise on account to members $1,183, terms n/30.12 Purchased golf shoes, sweaters, and other accessories on account from Birdie Sportswear

$1,080, terms 1/10, n/3014 Paid Ryder Co. in full.17 Received credit from Birdie Sportswear for merchandise returned $280.20 Made sales on account to members $1,055, terms n/30.21 Paid Birdie Sportswear in full.27 Granted an allowance to members for clothing that did not fit properly $90.30 Received payments on account from members $1,427.

Your answer is correct.

   

53

Sales 1905

Cost of Goods Sold 762

Merchandise Inventory 762

Sales Returns and Allow ances 195

Accounts Receivable 195

Merchandise Inventory 78

Cost of Goods Sold 78

Page 54: Accounting and Finance

Journalize the April transactions using a periodic inventory system. (List multiple debit/credit entries from largest to smallest amount, e.g. 10, 5, 2. Round answers to the nearest whole dollar, e.g. 5,275.)

Date Account/Description Debit Credit

Apr. 5  

           

 

Apr. 7  

           

 

Apr. 9  

           

 

Apr. 10  

         

 

Apr. 12  

           

 

Apr. 14  

           

 

           

 

Apr. 17  

           

 

Apr. 20  

           

 

Apr. 21  

           

 

           

 

Apr. 27  

           

 

Apr. 30  

           

 

54

Purchases 1942

Accounts Payable 1942

Freight-in 116

Cash 116

Accounts Payable 270

Purchase Returns and Allow ances 270

Accounts Receivable 1183

Sales 1183

Purchases 1080

Accounts Payable 1080

Accounts Payable 1672

Cash 1621.84

Purchase Discounts 50.16

Accounts Payable 280

Purchase Returns and Allow ances 280

Accounts Receivable 1055

Sales 1055

Accounts Payable 800

Cash 792

Purchase Discounts 8.00

Sales Returns and Allow ances 90

Accounts Receivable 90

Cash 1427

Accounts Receivable 1427

Page 55: Accounting and Finance

Your answer is correct.

   Using T accounts, enter the beginning balances in the ledger accounts and post the April transactions. Complete the trial balance on April 30, 2012 below. (If answer is zero please enter 0, do not leave any fields blank.)

THOUSAND OAKS PRO SHOPTrial BalanceApril 30, 2012

Debit Credit

Cash$ $

Accounts Receivable

Merchandise Inventory

Accounts Payable

Common Stock

Sales

Sales Returns and Allowances

Purchases

Purchase Returns and Allowances

Purchase Discounts

Freight-in

$ $

Your answer is correct.

   Complete the income statement through Gross Profit below, assuming merchandise inventory on hand at April 30 is $5,999. (List multiple entries from largest to smallest amounts, e.g. 10, 5, 1. Enter all amounts as positive amounts and subtract where necessary.)

THOUSAND OAKS PRO SHOPIncome Statement (Partial)

For the Month Ended April 30, 2012Sales revenues  

       

      Less:  

55

Follow the same calculations on page 50.

1954.16 0

721 0

4057 0

0 0

0 7114

0 2238

90 0

3022 0

0 550

0 58.16

116 0

9960.16 9960.16

Sales

Sales Returns and Allow ances

Page 56: Accounting and Finance

      Net Sales  

Cost of Goods Sold  

       

       

$

      Less:  $

               

      Net purchases  

      Add:  

      Cost of goods purchased  

      Cost of goods available for sale  

       

          Cost of goods sold  

      Gross Profit  

Test 1

View Attempt 1 of 1

Title: Test #1 September 4, 2012

Started: September 4, 2012 10:04 AM

Submitted: September 4, 2012 1:05 PM

Time spent: 03:00:51

56

Inventory, April 1

Purchases 3022

Purchase returns & all. 550

Purchase discounts58.16 608.12

2413.88

Freight-in116

Inventory, April 30

Page 57: Accounting and Finance

Total score: 82.47/100 = 82.47% Total score adjusted by 12.0 Maximum possible score: 100

1.

 

At the beginning of the year, assets totaled $400,000 and liabilities totaled $150,000.During the year, the company had $50,000 in revenues, paid $5000 in dividends, had an increase in assets of $200,000 and expenses of $55000. What was the ending balance for liabilities at the end of the year?

Student Response:

600,000 = 350,000 - 5,000 + 50,000 - 55,000

Sample Correct Answer

Assets=400000+200000=600000 SE=250000+50000-5000-55000=240000 Liabilities=360000

Score: 0/5

2.

  Special Delivery was started on May 1 with an investment of $45,000 cash. Following are the assets and liabilities of the company on May 31, 20XX, and the revenues and expenses for the month of May, its first month of operations.

Accounts receivable $ 6,200 Notes payable $28,000

Service revenue 10,400 Salaries and wages expense 2,000

Advertising expense 800 Equipment 56,000

Accounts payable 2,400 Maintenance and repairs expense

2,900

Cash 15,800 Insurance expense 400

No additional common stock was issued in May, but a dividend of $1,700 in cash was paid.

Instructions

(a) Prepare an income statement for the month of May.

57

Page 58: Accounting and Finance

Student Response:

Income Statement For the Month Ended in May 31, 2012 Revenues Service Revenue 10, 400 Expenses Maintenance and repairs expense - 2,900 Salaries and wages expense - 2,000 Advertising expense - 800 Insurance expense - 400 Total Expenses - - 6100 --------------- Net Income - 4300

Sample Correct Answer

Special DeliveryIncome StatementFor the month ended May 31, 20XXRevenue:Service Revenue...........................................10400Expenses:Advertising..........................800Salaries and Wages Exp.......2000Maint. & Repair Exp.............2900Insurance Expense...............400Total Expenses..............................................6100Net Income...................................................43000

Score: 4.67/5

3.

  The following are the major balance sheet classifications.

Current assets (CA) Current liabilities (CL)

Long-term investments (LTI) Long-term liabilities (LTL)

Property, plant, and equipment (PPE)

Stockholders' equity (SE)

Intangible assets (IA)

Instructions

Classify each of the following financial statement items taken from Inshore Corporation's

58

Page 59: Accounting and Finance

balance sheet.

________ Prepaid advertising

________ Equipment

________ Trademarks

________ Salaries and wages payable

________ Income taxes payable

________ Retained earnings

________ Accounts receivable

________ Land held for future use

________ Patents

________ Bonds payable

________ Common stock

________ Accumulated depreciation—equipment

________ Unearned sales revenue

________ Inventory

Student Response:

___CA_____ Prepaid advertising ____PPE____ Equipment ____IA____ Trademarks _____CL___ Salaries and wages payable ____CL____ Income taxes payable ____SE____ Retained earnings ____CA____ Accounts receivable _____LTI___ Land held for future use ___IA_____ Patents ____LTL____ Bonds payable ____SE____ Common stock ____PPE____ Accumulated depreciation—equipment ____CL____ Unearned sales revenue ____CA____ Inventory

Sample Correct Answer

__CA______ Prepaid advertising __PPE______ Equipment _IA______ Trademarks _CL_______ Salaries and wages payable ___CL_____ Income taxes payable ___SE_____ Retained earnings ___CA_____ Accounts receivable __LTI______ Land held for future use __IA______ Patents __LTL______ Bonds payable __SE______ Common stock __PPE______ Accumulated depreciation—equipment ___CL_____ Unearned sales revenue ___CA_____ Inventory

59

Page 60: Accounting and Finance

Score: 12/12

4.

 

For each item below, indicate whether a debit (DR) or credit (CR) applies.

1. Decrease in Trademark2. Increase in Accumulated Depreciation3. Decrease in Unearned Revenues4. Increase in Dividends5. Decrease in Interest Receivable6. Increase in Depreciation Expense7. Decrease in Mortgage Payable8. Increase in Supplies9. Increase in Wages Expense10. Decrease in Notes Receivable

Student Response:

1.CR 2.DR 3.DR 4.DR 5.CR 6.DR 7.DR 8.DR 9.DR 10.CR

Sample Correct Answer

1.CR 2.CR 3.DR 4.DR 5.CR 6.DR 7.DR 8.DR 9.DR 10.CR

Score: 10.8/12

5.

  Analyze and record the following transactions for Marshland Company in a general journal for the month of June.

June 1 Stockholders invested $25,000 cash in exchange for common stock to start a business.

June 3 Purchased equipment for $4,000 paying $2,000 in cash and the remainder due in 30 days.

June 7 Purchased supplies for $1,200 cash.

60

Page 61: Accounting and Finance

June 8 Received a bill for $200 for advertising.

June 13 Cash receipts from customers for pizza sales amounted to $1,500.

June 15 Paid salaries of $200 to workers.

June 20 Received a deposit of $100 from a customer before starting production.

June 21 Paid $200 for advertising that was previously billed on June 8th.

June 24 Stockholders were paid dividends of $800.

June 28 Incurred utility expenses for month on account, $200.

Marshland's chart of accounts includes: Accounts Payable, Accounts Receivable, Advertising Expense, Cash, Common Stock, Dividends, Equipment, Salaries Expense, Sales Revenue, Supplies, Supplies Expense, Unearned Revenue, and Utility Expense.

Student Response:

Date Account/Description Debit/Credit June 1 Cash/Common Stock 25,000/25,000 June 3 Equipment/ 4,000/0 Cash/ 0/2,000 Accounts Payable 0/2,000 June 7 Cash/Supplies 1,200/1,200 June 8 Advertising Expense/Accounts Receivable 200/200 June 13 Sales Revenue/Cash 1,500/1,500 June 15 Salaries Expense/Cash 200/200 June 20 Cash/Unearned Revenue 100/100 June 21 Accounts Payable/Cash 200/100 June 24 Accounts Payable/Dividends 800/800 June 28 Utility Expenses/Cash 200/200

Sample Correct Answer

June 1 Cash 25000 Common Stock 25000 3 Equipment 4000 Cash 2000 Accounts Payable 2000 7 Supplies 1200 Cash 1200 8 Advertising Expense 200 Accounts Payable 200 13 Cash 1500 Revenue 1500 15 Salaries Expense 200 Cash 200 20 Cash 100 Unearned Revenue 100 21 Accounts Payable 200 Cash 200 24 Dividends 800 Cash 800 28 Utilities Expense 200 Accounts Payable 200

Score: 13/21

6.

  One of the accounting concepts upon which adjustments for prepayments and accruals are based is:

61

Page 62: Accounting and Finance

Student Response Value Correct Answer Feedback

A. cost

B. monetary unit

C. expense recognition 100%

D. economic entity

Score: 3/3

7.

  The revenue recognition principle dictates the revenue should be recognized in the accounting records

Student Response Value Correct Answer Feedback

A. when cash is received.

B. in the period that income taxes are paid.

C. at the end of the month.

D. when it is earned. 100%

62

Page 63: Accounting and Finance

Score: 3/3

8.

 

On January 1, 2011, M. Johanson Company purchased equipment for $30,000. The company is depreciating the equipment at the rate of $500 per month. The book value of the equipment at December 31, 2011 is:

Student Response Value Correct Answer Feedback

A. $24,000 100%

B. $0

C. $6,000

D. $30,000

Score: 3/3

9.

  Otto's Tune-Up Shop follows the revenue recognition principle. Otto services a car on August 31. The customer picks up the vehicle on September 1 and mails the payment to Otto on September 5. Otto receives the check in the mail on September 6. When should Otto show that the revenue was earned?

Student Response Value Correct Answer Feedback

A. August 31 100%

63

Page 64: Accounting and Finance

B. September 6

C. August 1

D. September 5

Score: 3/3

10.

 

The primary difference between prepaid and accrued expenses is that prepaid expenses have

Student Response Value Correct Answer Feedback

A. not been paid and accrued expenses have.

B. not been recorded and accrued expenses have.

C. been recorded and accrued expenses have not.

100%

D. been incurred and accrued expenses have not.

Score: 3/3

11.

64

Page 65: Accounting and Finance

 

Adjusting entries are required

Student Response Value Correct Answer Feedback

A. because some costs expire with the passage of time and have not yet been journalized.

B. when the company's profits are below the budget.

C. when expenses are recorded in the period in which they are earned.

D. None of the above. 0%

Score: 0/3

12.

  An adjusting entry:

Student Response Value Correct Answer Feedback

A. is always a compound entry.

B. affects a balance sheet account and an income statement account.

100%

65

Page 66: Accounting and Finance

C. affects two balance sheet accounts.

D. affects two income statement accounts.

Score: 3/3

13.

 

A law firm received $2000 cash for legal services to be rendered in the future the full amount was credited to a liability account Unearned Service Revenue. If the legal services have been rendered at the end of the accounting period and no adjusting entry is made, this would cause

Student Response Value Correct Answer Feedback

A. expenses to be overstated.

B. liabilities to be understated.

C. revenues to be understated.

D. net income to be overstated.

0%

Score: 0/3

14.

  Greese Company purchased office supplies costing $4,000 and debited Office Supplies for the full amount. At the end of the accounting period, a physical count of office supplies

66

Page 67: Accounting and Finance

revealed $1,100 still on hand. The appropriate adjusting journal entry to be made at the end of the period would be:

Student Response Value Correct Answer Feedback

A. debit Office Supplies Expense, $2,900; credit Office Supplies, $2,900.

100%

B. debit Office Supplies, $2,900; credit Office Supplies Expense, $2,900.

C. debit Office Supplies, $1,100; credit Office Supplies Expense, $1,100.

D. debit Office Supplies Expense, $1,100; credit Office Supplies, $1,100.

Score: 3/3

15.

  On July 1 the Fisher Shoe Store paid $15,000 to Acme Realty for 6 months rent beginning July 1. Prepaid Rent was debited for the full amount. If financial statements are prepared on July 31, the adjusting entry to be made by the Fisher Shoe Store is:

Student Response Value Correct Answer Feedback

A. debit Prepaid Rent, $2,500; credit Rent

0%

67

Page 68: Accounting and Finance

Expense, $2,500.

B. debit Rent Expense, $15,000; credit Prepaid Rent, $12,500.

C. debit Rent Expense, $15,000; credit Prepaid Rent, $2,500.

D. debit Rent Expense, $2,500; credit Prepaid Rent, $2,500.

Score: 0/3

16.

  An accumulated depreciation account

Student Response Value Correct Answer Feedback

A. is offset against total assets on the balance sheet.

B. is a contra liability account.

C. has a normal credit balance.

D. increases on the debit side.

0%

68

Page 69: Accounting and Finance

Score: 0/3

17.

 

Which of the statements below is NOT true?

Student Response Value Correct Answer Feedback

A. An adjusted trial balance prove the mathematical equality of debits and credits in the ledger.

B. An adjusted trial balance is prepared before all transactions have been journalized.

C. An adjusted trial balance should show ledger account balances.

D. An adjusted trial balance can be used to prepare financial statements.

0%

Score: 0/3

18.

  The following accounts show balances on the adjusted trial balance. Which of these account

69

Page 70: Accounting and Finance

balances will NOT appears the same on the balance sheet?

Student Response Value Correct Answer Feedback

A. Retained Earnings 100%

B. Notes Payable

C. Accounts Receivable

D. Common Stock

Score: 3/3

19.

 

The closing entry process consists of closing

Student Response Value Correct Answer Feedback

A. out the Retained Earnings account.

B. all asset and liability accounts.

C. all temporary accounts.

100%

D. all permanent accounts.

Score: 3/3

70

Page 71: Accounting and Finance

20.

 

At the end of the fiscal year, the usual adjusting entry for accrued salaries owed to employees was omitted. Which of the following statements is true?

Student Response Value Correct Answer Feedback

A. Assets at the end of the year are understated.

B. Stockholders' equity at the end of the year is understated.

C. Salary Expense for the year is overstated.

D. Liabilities at the end of the year are understated.

100%

Score: 3/3

 

Jump to Navigation Frame Jump to Content Frame

   Your location: Learning Modules › Chapter 5 › Quiz 5 › >Quiz 5 › Assessments › View All Submissions › View Attempt

View Attempt 1 of 1

Title: >Quiz 5

71

Page 72: Accounting and Finance

Started: September 7, 2012 10:31 AM

Submitted:

September 7, 2012 11:07 AM

Time spent:

00:35:57

Total score:

100/100 = 100% Total score adjusted by 0.0 Maximum possible score: 100

1.

 

Net income will result if gross profit exceeds

Student Response

A. cost of goods sold.

B. cost of goods sold plus operating expenses.

C. purchases.

D. operating expenses.

Score: 10/10

2.

  The journal entry to record a return of merchandise purchased on account under a perptual inventory system would credit

Student Response

A. Accounts Payable.

72

Page 73: Accounting and Finance

B. Sales.

C. Purchase Returns and Allowances.

D. Merchandise Inventory.

Score: 10/10

3.

 

Hunter Company purchased merchandise inventory with an invoice price of $6,000 and credit terms of 2/10, n/30. What is the net cost of the goods if Hunter Company pays within the discount period?

Student Response

A. $5,520

B. $5,880

C. $6,000

D. $5,400

Score: 10/10

4.

  Freight costs incurred by a seller on merchandise sold to customers will cause an increase

Student Response

73

Page 74: Accounting and Finance

A. to a contra-revenue account of the seller.

B. to the cost of goods sold of the seller.

C. in the selling expenses of the buyer.

D. in operating expenses for the seller.

Score: 10/10

5.

 

If a company is given credit terms of 2/10, n/30, it should

Student Response

A. pay within the credit period buy don't take the trouble to invest the cash while waiting to pay the bill.

B. hold off paying the bill until the end of the credit period, while investing the money at 10% annual interest during this time.

C. recognize that the supplier is desperate for cash and withhold payment until the end of the credit period while negotiating a lower sales price.

D. pay within the discount period and recognize a savings.

Score: 10/10

6.

  Under the perpetual inventory system, in addition to making the entry to record a sale, a company would

Student Response

74

Page 75: Accounting and Finance

A. debit Cost of Goods Sold and credit Purchases.

B. debit Merchandise Inventory and credit Cost of Goods Sold.

C. debit Cost of Goods Sold and credit Merchandise Inventory.

D. make no additional entry until the end of the period.

Score: 10/10

7.

 

The credit terms offered to a customer by a business firm were 2/10,n/30, which means

Student Response

A. the customer must pay the bill within 10 days.

B. two sales returns can be made within 10 days of the invoice date and no returns thereafter.

C. the customer can deduct a 2% discount if the bill is paid between the 10th and 30th day from the invoice date.

D. the customer can deduct a 2% discount if the bill is paid within 10 days of the invoice date.

Score: 10/10

8.

  Ellis Comapny sells merchandise on account for $1,500 to Thomas Company with credit terms of 2/10, n/30. Thomas Company returns $500 of merchandise that was damaged, along with a check to settle the account within the discount period. What entry does Ellis Company make upon receipt of the check?

Student Response

75

Page 76: Accounting and Finance

A. debit Cash, $1,000; credit Accounts Receivable $1,000

B. debit Cash, $1,470; debit Sales Discount, $30; credit Sales Returns and Allowances, $500; credit Accounts Receivable $1,000

C. debit Cash, $980; debit Sales Returns and Allowances, $500; debit Sales Discounts, $20; credit Accounts Receivable $1,500

D. debit Cash, $980; debit Sales Returns and Allowances, $520; credit Accounts Receivable, $1,000

Score: 10/10

9.

 

The operating expenses section of an income statement for a merchandising company would not include

Student Response

A. Cost of Goods Sold.

B. Insurance Expense.

C. Freight-Out.

D. Utilities Expense.

Score: 10/10

10.

76

Page 77: Accounting and Finance

 

Financial information is presented below:

Operating Expenses $45,000Sales Returns and Allowances $13,000Sales Discount $6,000Sales $150,000Cost of Goods Sold $77,000

Gross Profit would be

Student Response

A. $76,000.

B. $54,000.

C. $73,000

D. $61,000.

Score: 10/10

 

77

13,000 + 6,000 = 19,000

150,000 – 19,000 = 131,000

131,000 – 77,000 = 54,000

Page 78: Accounting and Finance

Chapter 6

AP6-1A

Your answer is correct.

   Kirk Limited is trying to determine the value of its ending inventory as of February 28, 2012, the company's year-end. The accountant counted everything that was in the warehouse, as of February 28, which resulted in an ending inventory valuation of $56,670. However, she didn't know how to treat the following transactions so she didn't record them.

For each of the following transactions, specify whether the item in question should be Included or Excluded in ending inventory, and if so, at what amount. (If excluded from inventory put 0 for the amount, note all boxes must be filled.)

    Included/Excluded Amount

(a)

On February 26, Kirk shipped to a customer goods costing $989. The goods were shipped FOB shipping point, and the receiving report indicates that the customer received the goods on March 2.

$

(b)

On February 26, Seller Inc. shipped goods to Kirk FOB destination. The invoice price was $522 plus $26 for freight. The receiving report indicates that the goods were received by Kirk on March 2.

$

(c)

Kirk had $613 of inventory at a customer's warehouse "on approval." The customer was going to let Kirk know whether it wanted the merchandise by the end of the week, March 4.

$

(d)Kirk also had $600 of inventory at a Balena craft shop, on consignment from Kirk. $

(e)

On February 26, Kirk ordered goods costing $839. The goods were shipped FOB shipping point on February 27. Kirk received the goods on March 1.

$

(f)

On February 28, Kirk packaged goods and had them ready for shipping to a customer FOB destination. The invoice price was $485 plus $43 for freight; the cost of the items was $300. The receiving report indicates that the goods were received by the customer on March 2.

$

(g)

Kirk had damaged goods set aside in the warehouse because they are no longer saleable. These goods originally cost $481 and, originally, Kirk expected to sell these items for $614.

$

Click here if you would like to Show Work for this question

78

Excluded 0

Excluded 0

Included 613

Included 600

Included 839

Included 300

Excluded 0

Page 79: Accounting and Finance

AP6-3A

Your answer is correct.

   Remsen Company Inc. had a beginning inventory of 240 units of Product MLN at a cost of $10 per unit. During the year, purchases were:

Feb. 20

842 units at $11 Aug. 12 481 units at $13

May 5 601 units at $12 Dec. 8 120 units at $14Remsen Company uses a periodic inventory system. Sales totaled 1,683 units.

(a) Determine the cost of goods available for sale.    

$(b) Determine the ending inventory and the cost of goods sold under each of the assumed cost

flow methods (FIFO, LIFO, and average cost). (For average cost round the per unit calculations to 3 decimal places, e.g. 3.425.  Round all answers to 0 decimal places, e.g. 2,500.)  FIFO LIFO Average Cost

Ending Inventory$ $ $

Cost of Goods Sold$ $ $

(c) Which cost flow method results in the lowest inventory amount for the balance sheet?    

     Which cost flow method results in the lowest cost of goods sold for the income statement?    

79

 

26807

7933 6371 7053.937

 

18874 20436 19753.063

LIFO

FIFO

User, 09/07/12,
Cost of good available for sale – average cost for ending inventory.
User, 09/07/12,
Cost of Goods Available for Sale – Ending Inventory = Cost of Goods Sold
User, 09/07/12,
Cost of Goods Available for Sale – Ending Inventory = Cost of Goods Sold
User, 09/07/12,
Cost of Goods Available for Sale/Total number of units purchased. This should give a decimal. Multiply decimal by (total number of units purchased – sales totaled units.)
User, 09/07/12,
Feb 20th Units – Aug 12th Units X Feb 20th unit price ($11) + Beginning Inventory Amount = Ending Inventory
User, 09/07/12,
Aug. 12 + Dec. 8 = Ending Inventory
Page 80: Accounting and Finance

Chapter 8

AP8-1A (a-d)

Sellmore.com uses the allowance method to estimate uncollectible accounts receivable. The company produced the following aging of the accounts receivable at year end.

    Number of Days Outstanding  Total 0-30 31-60 61-90 91-120 Over 120

Accounts receivable $465,400 $267,400 $96,400 $66,600 $14,000 $21,000% uncollectible   1% 4% 5% 6% 10%

Your answer is correct.

   Calculate the total estimated bad debts based on the above information.

    Number of Days Outstanding  Total 0-30 31-60 61-90 91-120 Over 120

Estimated bad debts

$ $ $ $ $ $

Your answer is correct.

   Prepare the year-end adjusting journal entry to record the bad debts using the aged uncollectible accounts receivable determined in the table above. Assume the unadjusted balance in the Allowance for Doubtful Accounts account is a $5,420 debit.

Account/Description Debit Credit

 

           

80

12800 2674 3856 3330 840 2100

Bad debts expense 18220

Allow ance for doubtful accounts 18220

User, 09/13/12,
Add the allowance for double accounts listed in the description to the estimated bad debts to get the total.
Page 81: Accounting and Finance

Your answer is correct.

   Of the above accounts, $5,540 is determined to be specifically uncollectible. Prepare the journal entry to write off the uncollectible account.

Account/Description Debit Credit

 

           

Your answer is correct.

   The company collects $5,540 subsequently on a specific account that had previously been determined to be uncollectible in the question above. Prepare the journal entry(ies) necessary to restore the account and record the cash collection.

Account/Description Debit Credit

 

           

(To restore the account.)    

 

           

(TO record the cash collection.)    

AP8-8A

Your answer is correct.

   Jander Company closes its books on July 31. On June 30 the Notes Receivable account balance is $24,090. Notes Receivable include the following.

Date Maker Face Value Term Maturity Date Interest RateMay 21 Allen Inc. $9,150   60 days July 20 8%May 25 Garnham Co. 10,560   60 days July 24 10%June 30 ERV Corp. 11,200   6 months December 31 9%During July the following transactions were completed.

July 5 Made sales of $5,180 on Jander credit cards.July 14 Made sales of $600 on Visa credit cards. The credit card service charge is 3%.

81

Allow ance for doubtful accounts 5540

Accounts receivable 5540

Accounts receivable 5540

Allow ance for doubtful accounts 5540

Cash 5540

Accounts receivable 5540

Page 82: Accounting and Finance

July 20 Received payment in full from Allen Inc. on the amount due.July 25 Received payment in full from Garnham Co. on the amount due.(a) Journalize the July transactions and the July 31 adjusting entry for accrued interest

receivable. Interest is computed using 360 days. (List multiple debit/credit entries from largest to smallest amount, e.g. 10, 5, 2.)

DateAccount/Description Debit Credit

July 5  

          

 

July 14  

   

          

 

July 20

          

 

          

 

July 24  

          

 

          

 

July 31  

          

 

(b) Enter the balances at July 1 in the receivable accounts and post the entries to all of the receivable accounts. (Use T accounts.)

Notes Receivable   Interest Receivable

 7/1 Bal.  7/20  

   7/31    

    7/24  

        

 7/31 Bal.     

  7/31 Bal.     

              Accounts Receivable         

 7/5    

        

 7/31 Bal.     

        

(c) Show the balance sheet presentation of the receivable accounts at July 31.Current assets  

      Notes receivable$

      Accounts receivable

      Interest receivable

         Total receivables$

82

 

Accounts receivable 5180

Sales 5180

Cash 582

Service charge expense 18

Sales 600

Cash 9272

Notes receivable 9150

Interest revenue 122

Cash 10736

Notes receivable 10560

Interest revenue 176

Interest receivable 84

Interest revenue 84

24090 9150 84

10560

4380 84

5180

5180

4380

5180

84

9644

 

 

User, 09/13/12,
Amount at the very top!
User, 09/13/12,
Face Value X Interest X Terms (will always be a fraction) = Interest Revenue
User, 09/13/12,
Face Value X Interest X Terms (will always be a fraction) = Interest Revenue
Page 83: Accounting and Finance

Jump to Navigation Frame Jump to Content Frame

   Your location: Learning Modules › Chapter 6 › Quiz 6 › >Quiz 6 › Assessments › View All Submissions › View Attempt

View Attempt 1 of 1

Title: >Quiz 6

Started: September 13, 2012 9:18 AM

Submitted:

September 13, 2012 10:17 AM

Time spent:

00:59:35

Total score:

30/100 = 30% Total score adjusted by 0.0 Maximum possible score: 100

1.

 

If goods in transit are shipped FOB destination

Student Response

A. no one has legal title tot he goods until they are delivered.

B. the seller has legal title to the goods until they are delivered.

C. the transportation company has legal title to the goods while the goods are in transit.

D. the buyer has legal title to the goods until they are delivered.

Score: 0/10

83

Page 84: Accounting and Finance

2.

 

Which of the following should not be included in the physical inventory of a company?

Student Response

A. Goods shipped on consignment to another company.

B. Goods in transit from another company shipped FOB shipping point.

C. All of the above should be included.

D. Goods held on consignment from another company.

Score: 10/10

3.

  July 1.......Beginning Inventory..........20 units @ $20 = $400July 7.......Purchases..........................70 units @ $21 = $1,470July 22.....Purchases..........................10 units @ $22 = $220Total cost of goods available........................................$2,090

A physical count of merchandise inventory on July 30 reveals that there are 35 units on hand.

Using the average cost method, the value of ending inventory is

Student Response

A. $735.00

B. $731.50

C. $770.00

84

Page 85: Accounting and Finance

D. $700.00

Score: 0/10

4.

 

July 1.......Beginning Inventory..........20 units @ $20 = $400July 7.......Purchases..........................70 units @ $21 = $1,470July 22.....Purchases..........................10 units @ $22 = $220Total cost of goods available........................................$2,090

A physical count of merchandise inventory on July 30 reveals that there are 35 units on hand.

Using the FIFO inventory method, the amount allocated to cost of goods sold for July is

Student Response

A. $1,390.

B. $1,345.

C. $1,380.

D. $1,425.

Score: 10/10

5.

  July 1.......Beginning Inventory..........20 units @ $20 = $400July 7.......Purchases..........................70 units @ $21 = $1,470July 22.....Purchases..........................10 units @ $22 = $220Total cost of goods available........................................$2,090

85

User, 09/13/12,
Same calculations used from the Comment followed below.
Page 86: Accounting and Finance

A physical count of merchandise inventory on July 30 reveals that there are 35 units on hand.

Using the FIFO inventory method, the amount allocated to ending inventory for July is

Student Response

A. $770.

B. $745.

C. $700.

D. $715.

Score: 10/10

6.

  Use the inventory information below for the month of July to answer the following question:

July 1 Beginning Inventory 20 units at $19 eachJuly 7 Purchase 70 units at $20 eachJuly 22 Purchase 10 units at $22 each

A physical count of merchandise inventory on July 31 reveals that there are 35 units on hand.

Using the LIFO inventory method, the amount allocated to COST OF GOODS SOLD for July is

Student Response

A. $680.

86

User, 09/13/12,
Use this number to subtract the units to give you the remain units needed to multiply by the July 7 unit value. Use all 20 units and have a remainder of 15 units. Multiply 15 units by $20.00 = 680. Add all units and their respect amounts to get the cost of goods for sale. Subtract that from 680 = LIFO Cost of Goods Sold.
Page 87: Accounting and Finance

B. $720.

C. $1,280.

D. $1,320.

Score: 0/10

7.

 

Use the inventory information below for the month of July to answer the following question:

July 1 Beginning Inventory 20 units at $19 eachJuly 7 Purchase 70 units at $20 eachJuly 22 Purchase 10 units at $22 each

A physical count of merchandise inventory on July 31 reveals that there are 35 units on hand.

Using the LIFO inventory method, the amount allocated to ENDING INVENTORY for July is

Student Response

A. $680.

B. $720.

C. $1,280.

D. $1,320.

Score: 0/10

87

Page 88: Accounting and Finance

8.

 

Which of the following statements is true regarding inventory cost flow assumptions?

Student Response

A. A company may use more than one costing method concurrently.

B. A company must comply with the method specified by industry standard.

C. A company must use the same method for domestic and foreign operations.

D. A company may never change its inventory costing method once it has chosen a method.

Score: 0/10

9.

 

Two companies report the same cost of goods available for sale but each employs a different inventory costing method. If the price of goods has increased during the period, then the company using

Student Response

A. LIFO will have the lowest cost of goods sold.

B. FIFO will have the highest cost of goods sold.

C. FIFO will have the highest ending inventory.

D. LIFO will have the highest ending inventory.

Score: 0/10

88

Page 89: Accounting and Finance

10.

 

Which inventory method generally results in costs allocated to ending inventory that will approximate their current cost?

Student Response

A. Whichever method that produces the highest ending inventory figure

B. FIFO

C. LIFO

D. Average Cost Method

Score: 0/10

 

89

Page 90: Accounting and Finance

   Your location: Learning Modules › Chapter 8 › Quiz 8 › >Quiz 8 › Assessments › View All Submissions › View Attempt

View Attempt 1 of 1

Title: >Quiz 8

Started: September 13, 2012 10:27 AM

Submitted:

September 13, 2012 11:07 AM

Time spent:

00:40:14

Total score:

90/100 = 90% Total score adjusted by 0.0 Maximum possible score: 100

1.

  The receivable that is usually evidenced by a formal instrument of credit is a(n)

Student Response

A. accounts receivable.

B. trade receivable.

C. notes receivable.

D. income tax receivable.

Score: 10/10

90

Page 91: Accounting and Finance

2.

 

Accounts receivable are valued and reported on the balance sheet

Student Response

A. only if they are not past due.

B. at gross amounts less sales returns and allowances.

C. in the investment section.

D. at cash realizable value.

Score: 10/10

3.

  Using the percentage of receivables method for recording bad debts expense, estimated uncollectible accounts are $25,000. If the balance of the Allowance for Doubtful Accounts is $8,000 debit before adjustment what is the balance after adjustment?

Student Response

A. $17,000

B. $25,000

C. $8,000

D. $33,000

91

Page 92: Accounting and Finance

Score: 10/10

4.

 

When an account becomes uncollectible and must be written off

Student Response

A. Sales should be debited.

B. Allowance for Doubtful Accounts should be credited.

C. Accounts Receivable should be credited.

D. Bad Debts Expense should be credited.

Score: 10/10

5.

  To record estimated uncollectible accounts using the allowance method, the adjusting entry would be a

Student Response

A. debit to Accounts Receivable and a credit to Allowance for Doubtful Accounts.

B. debit to Allowance for Doubtful Accounts and a credit to Accounts Receivable.

C. debit to Bad Debts Expense and a credit to Allowance for Doubtful Accounts.

D. debit to Loss on Credit Sales and a credit to Accounts Receivable.

Score: 10/10

92

Page 93: Accounting and Finance

6.

 

The balance in Allowance for Doubtful Accounts must be considered prior to end of period adjustment when using which of the following methods?

Student Response

A. Allowance method

B. Accrual method

C. Direct write-off method

D. Net realizable method

Score: 10/10

7.

  If an account is collected after having been previously written off

Student Response

A. there will be both a debit and a credit to accounts receivable.

B. both income statement and balance sheet accounts will be affected.

C. the allowance account should be debited.

D. only the control account needs to be credited.

Score: 10/10

93

Page 94: Accounting and Finance

8.

 

Under the allowance method, Bad Debt Expense is recorded

Student Response

A. several times during the accounting period.

B. when the loss amount is known.

C. for an amount that the company estimates is will not collect.

D. when an individual account is written off.

Score: 10/10

9.

  Using the percentage of receivables method for recording bad debts expense, estimated uncollectible accounts are $25,000. If the balance of the Allowance for Doubtful Accounts is $8,000 debit before adjustment what is the amount of bad debt expense for that period?

Student Response

A. $8,000

B. $25,000

C. $33,000

D. $17,000

94

Page 95: Accounting and Finance

Score: 0/10

10.

 

An aging of a company's accounts receivable indicates that $3,000 are estimated to be uncollectible. If Allowance for Doubtful Accounts has a $1,200 credit balance, the adjustment to record bad debts for the period will require a

Student Response

A. a debit to Allowance for Doubtful Accounts for $1,800.

B. credit to Allowance for Doubtful Accounts for $3,000.

C. debit to Bad Debts Expense for $3,000.

D. debit to Bad Debts Expense for $1,800.

Score: 10/10

95

Page 96: Accounting and Finance

Chapter 9

AE9-18

Your answer is correct.

   Buckeye Bus Lines uses the units-of-activity method in depreciating its buses. One bus was purchased on January 1, 2012, at a cost of $172,720. Over its 4-year useful life, the bus is expected to be driven 203,200 miles. Salvage value is expected to be $10,160.

(a) Compute the depreciation cost per unit. (Round answer to 2 decimal places, e.g. 0.20)    

$  per mile(b) Prepare a depreciation schedule assuming actual mileage was: 2012, 50,800; 2013, 66,040; 2014,

52,070; and 2015, 34,290. (Round depreciation cost per unit to 2 decimal places, e.g. 0.20 and all other answers to 0 decimal places, e.g. 25,330.)

Computation End of Year

Years Units of Activity ×

Depreciation Cost/Unit =

Annual Depreciation

ExpenseAccumulated Depreciation Book Value

2012    

2013    

2014    

2015    

Show Work is REQUIRED for this question; access the WhiteBoard application by clicking this link

AP9-8A

Your answer is correct.

   Rogers Corporation purchased machinery on January 1, 2012, at a cost of $326,100. The estimated useful life of the machinery is 4 years, with an estimated residual value at the end of that period of $14,700. The company is considering different depreciation methods that could be used for financial

96

.80

 

50800 .80 40640 40640 132080

66040 .80 52832 93472 79248

52070 .80 41656 135128 37592

34290 .80 27432 162560 10160

User, 09/14/12,
132,080 – 52,832 = 79,248
User, 09/14/12,
40,640 + 52,832 = 93, 472
User, 09/14/12,
172,720 – 40640 = 132,080
User, 09/14/12,
172,720 – 10,160 = 162,560, then /203,200 = .80 MUST REMEMBER TO HAVE 2 DECIMAL PLACES!
Page 97: Accounting and Finance

reporting purposes.

(a) Prepare separate depreciation schedules for the machinery using the straight-line method, and the declining-balance method using double the straight-line rate. (Round answers to 0 decimal places, e.g. 15 or 15%.)

STRAIGHT-LINE DEPRECIATIONComputation   End of Year

YearsDepreciable

Cost ×Depreciation

Rate =

AnnualDepreciation

ExpenseAccumulated Depreciation Book Value

2012$

 %

 $ $ $

2013  %

 

2014  %

 

2015  %

 

DOUBLE-DECLINING-BALANCE DEPRECIATIONComputation   End of Year

Years

Book Value Beginning of

Year ×Depreciation

Rate =

AnnualDepreciation

ExpenseAccumulated Depreciation Book Value

2012$

 %

 $ $ $

2013  %

 

2014  %

 

2015  %

  *26,062

* Adjusted so ending book value will equal salvage value.

(b) Which method would result in the higher reported 2012 income?    

     In the highest total reported income over the 4-year period?    

(c) Which method would result in the lower reported 2012 income?    

     In the lowest total reported income over the 4-year period?    

AP9-3A

97

311400 25 77850 77850 248250

311400 25 77850 155700 170400

311400 25 77850 233550 92550

311400 25 77850 311400 14700

326100 50 163050 163050 163050

163050 50 81525 244575 81525

81525 50 40763 285338 40762

40762 50 311400 14700

Straight-line

Both

Double-declining balance

Both

User, 09/15/12,
326,100 – 81,525 = 163,050. Book Value Beginning of Year – Annual Depreciation Expense = Book Value….etc…for the upcoming year.
User, 09/15/12,
Double the depreciation rate from the straight-line method.
Page 98: Accounting and Finance

Your answer is incorrect.

Presented here are selected transactions for Snow Company for 2012.

Jan. 1 Retired a piece of machinery that was purchased on January 1, 2002. The machine cost $115,730 on that date and had a useful life of 10 years with no salvage value.

June 30 Sold a computer that was purchased on January 1, 2009. The computer cost $48,900 and had a useful life of 5 years with no salvage value. The computer was sold for $16,300.

Dec. 31 Discarded a delivery truck that was purchased on January 1, 2007. The truck cost $50,530 and was depreciated based on an 8-year useful life with a $4,890 salvage value.

Journalize all entries required on the above dates, including entries to update depreciation on assets disposed of, where applicable. Snow Company uses straight-line depreciation. (Assume depreciation is up to date as of December 31, 2011.) (List multiple debit/credit entries from largest to smallest amount, e.g. 10, 5, 2. Round answers to 0 decimal places, e.g. 2,550.)

Date Account/Description Debit Credit

Jan. 1  

       

 

June 30  

       

 

  (To record depreciation on computer.)    

   

   

       

 

          

 

  (To record the sale of the computer.)    

Dec. 31  

       

 

  (To record depreciation on truck.)    

98

Accumulated depreciation - machinery 115730

Machinery 115730

Depreciation expense

Accumulated depreciation - computer

Accumulated depreciation - computer 34230

Cash 16300

Computer 48900

Gain on disposal 1630

Depreciation expense 5705

Accumulated depreciation - truck 5705

Page 99: Accounting and Finance

   

   

       

 

  (To record disposal of truck.)    

Student did not submit Show Work for this question

Jump to Navigation Frame Jump to Content Frame

   Your location: Learning Modules › Chapter 9 › Quiz 9 › >Quiz 9 › Assessments › View All Submissions › View Attempt

View Attempt 1 of 1

Title: >Quiz 9

Started: September 17, 2012 10:38 AM

Submitted:

September 17, 2012 11:32 AM

Time spent:

00:53:44

Total score:

70/100 = 70% Total score adjusted by 0.0 Maximum possible score: 100

1.

  A company purchased land for $70,000 cash. Real estate brokers' commission was $5,000 and $7,000 was spent for demolishing an old building on the land before construction of a new building could start. Under the cost principle, the cost of land would be recorded at

Student Response

A. $70,000

99

Accumulated depreciation - truck 34230

Loss on disposal 16300

Delivery Truck 50530

Page 100: Accounting and Finance

B. $82,000

C. $75,000

D. $77,000

Score: 10/10

2.

 

Recording depreciation each period is necessary in accordance with the

Student Response

A. cost principle.

B. asset valuation principle.

C. matching principle.

D. going concern principle.

Score: 10/10

3.

  In computing depreciation, salvage value is

Student Response

A. ignored in all the depreciation methods.

B. the fair market value of a plant asset on the date of acquisition.

100

Page 101: Accounting and Finance

C. an estimate of a plant asset's value at the end of its useful life.

D. subtracted from accumulated depreciation to determine the plant asset's depreciable cost.

Score: 10/10

4.

 

Equipment with a cost of $192,000 has an estimated salvage value of $18,000 and an estimated life of 4 years or 12,000 hours. It is to be depreciated by the straight-line method. What is the amount of depreciation for the first full year, during which the equipment was used 3,300 hours?

Student Response

A. $43,500

B. $49,500

C. $48,000

D. $52,500

Score: 10/10

5.

  A truck was purchased for $15,000 and it was estimated to have a $3,000 salvage value at the end of its useful life. Monthly depreciation expense of $200 was recorded using the straight-line method. The annual depreciation rate is

Student Response

A. 2%.

101

Page 102: Accounting and Finance

B. 25%.

C. 16%.

D. 20%.

Score: 0/10

6.

 

Which of the following methods of computing depreciation is production based?

Student Response

A. units-of-activity

B. straight-line

C. none of the above

D. declining-balance

Score: 10/10

7.

  On January 1, a machine with a useful life of four years and a residual value of $3,000 was purchased for $19,000. What is the depreciation expense for year 2 under straight-line depreciation?

Student Response

102

Page 103: Accounting and Finance

A. $4,750

B. $4,000

C. $2,000

D. $8,000

Score: 10/10

8.

 

Bates Company purchased equipment on January 1, 2006, at a total invoice cost of $600,000. The equipment has an estimated salvage value of $15,000 and an estimated useful life of 5 years. What is the amount of accumulated depreciation at December 31, 2007, if the straight-line method of depreciation is used?

Student Response

A. $240,000

B. $120,000

C. $117,000 x 2 = D. 234,000

D. $234,000

Score: 0/10

9.

  A machine that was purchased on January 1 for $15,000 has an estimated salvage value of $3,000. If the machine's depreciation rate is 20%, its annual depreciation is

103

Page 104: Accounting and Finance

Student Response

A. $2,400.

B. $12,000.

C. $3,000.

D. $3,600.

Score: 10/10

10.

 

On October 1, 2007, Colt Company places a new asset into service. The cost of the asset is $40,000 with an estimated 5-year life and $10,000 salvage value at the end of its useful life. What is the book value of the plant asset on the December 31, 2007 balance sheet assuming that Colt Company uses the double-declining method of depreciation?

Student Response

A. $26,000

B. $38,000

C. $30,000

D. $36,000

Score: 0/10

 

104

Page 105: Accounting and Finance

Sample Test Review

AE5-4

On June 10 Harris Company purchased $8,434 of merchandise from Goetz Company, terms 6/10, n/30. Harris pays the freight costs of $438 on June 11. Damaged goods totaling $574 are returned to Goetz for credit on June 12. On June 19 Harris Company pays Goetz Company in full, less the purchase discount. Both companies use a perpetual inventory system.

Your answer is correct.

   Prepare separate entries for each transaction on the books of Harris Company.  (List multiple debit/credit entries from largest to smallest amount, e.g. 10, 5, 2. Round all answers to 0 decimal places, e.g. 2.555.)

Date Account/Description Debit Credit

June 10  

           

 

June 11  

           

 

June 12  

           

 

June 19  

            

 

         

 

105

Merchandise Inventory 8434

Accounts Payable 8434

Merchandise Inventory 438

Cash 438

Accounts Payable 574

Merchandise Inventory 574

Accounts Payable 7860

Cash 7388.4

Merchandise Inventory 471.60

User, 09/18/12,
8,434 x 6/10 (terms) = 4716, move 1 decimal place to the left = 471.6
User, 09/18/12,
Accounts Payable – Merchandise Inventory = Cash
User, 09/18/12,
8,434 – 438 (freight cost) = Accounts Payable.
Page 106: Accounting and Finance

Your answer is correct.

   Prepare separate entries for each transaction for Goetz Company. The merchandise purchased by Harris on June 10 cost Goetz $2,178, and the goods returned cost Goetz $225. (If no entry is required write No Entry for the account and 0 for the amount. List multiple debit/credit entries from largest to smallest amount, e.g. 10, 5, 2. Round all answers to 0 decimal places, e.g. 2,555.)

Date Account/Description Debit Credit

June 10  

           

 

  (To record the sale)    

   

           

 

  (To record the cost of inventory)    

June 11  

           

 

June 12  

         

 

  (To record return of damaged goods)    

   

           

 

  (To adjust inventory for the returned goods)    

June 19  

   

           

 

AE5-13

106

Record numbers from June 19

Accounts Receivable 8434

Sales 8434

Cost of Goods Sold 2178

Merchandise Inventory 2178

No Entry 0

No Entry 0

Sales Returns and Allow ances 574

Accounts Receivable 574

Merchandise Inventory 225

Cost of Goods Sold 225

Cash 7388.4

Sales Discounts 471.60

Accounts Receivable 7860

Page 107: Accounting and Finance

Your answer is correct.

   This information relates to Edyburn Co.

1. On April 5 purchased merchandise from Hansen Company for $28,300, terms 2/10, net/30.2. On April 6 paid freight costs of $1,487 on merchandise purchased from Hansen Company.3. On April 7 purchased equipment on account for $30,200.4. On April 8 returned some of April 5 merchandise to Hansen Company which cost $4,700.5. On April 15 paid the amount due to Hansen Company in full.(a) Prepare the journal entries to record these transactions on the books of Edyburn o. using a

periodic inventory system. (List multiple debit/credit entries from largest to smallest amount, e.g. 10, 5, 2.)

DateAccount/Description Debit Credit

April 5  

           

 

April 6  

           

 

April 7  

           

 

April 8  

         

 

April 15

 

           

 

           

 

(b) Assume that Edyburn Co. paid the balance due to Hansen Company on May 4 instead of April 15. Prepare the journal entry to record this payment.

DateAccount/Description Debit Credit

May 4  

           

 

AE6-1

Your answer is correct.

   

Worthmore Bank and Trust is considering giving Madsen Company a loan. Before doing so, they decide that further discussions with Madsen's accountant may be desirable. One area of particular concern is the inventory account, which has a year-end balance of $281,522. Discussions with the accountant reveal the following.

107

Same calculations from previous page.

Purchases 28300

 

Accounts Payable 28300

Freight-in 1487

Cash 1487

Equipment 30200

Accounts Payable 30200

Accounts Payable 4700

Purchase Returns and Allow ances 4700

Accounts Payable

 

23600

Cash 23128

Purchase Discounts 472

Accounts Payable 23600

Cash 23600

Page 108: Accounting and Finance

Determine the correct inventory amount on December 31, by filling in the beginning inventory and adjustments for each item listed below to arrive at corrected inventory. (If no effect input 0 for the amount and if the amount should be deducted enter it with either a negative sign preceding the number, e.g. -45 or parenthesis, e.g. (45). Note all boxes must be filled.)

Ending inventory - physical count$

1. Madsen sold goods costing $56,627 to Allen Company FOB shipping point on December 28. The goods are not expected to reach Allen until January 12. The goods were not included in the physical inventory because they were not in the warehouse.

2. The physical count of the inventory did not include goods costing $104,195 that were shipped to Madsen FOB destination on December 27 and were still in transit at year-end.

3. Madsen received goods costing $27,350 on January 2. The goods were shipped FOB shipping point on December 26 by Lynch Co. The goods were not included in the physical count.

4. Madsen sold goods costing $51,196 to Finet of Canada FOB destination on December 30. The goods were received in Canada on January 8. They were not included in Madsen's physical inventory.

5. Madsen received goods costing $39,323 on January 2 that were shipped FOB destination on December 29. The shipment was a rush order that was supposed to arrive December 31. This purchase was included in the ending inventory of $281,522.Correct inventory

$

Click here if you would like to Show Work for this question

AE6-5

Your answer is correct.

   

Klumb Inc. uses a periodic inventory system. Its records show the following for the month of May, in which 96 units were sold.

DateExplanation Units Unit Cost Total Cost

May 1 Inventory 37 $11 $407  May 15 Purchase 31 12 372  May 24 Purchase 49 13 637  

Total 117 $1,416  

Calculate the ending inventory at May 31 using the (a) FIFO, (b) average-cost, and (c) LIFO methods. (Round all answers to 0 decimal places, e.g. 2,555. For average cost computations round the per unit cost to 3 decimal places, e.g. 2.550.)

FIFO$

Average-cost$

LIFO$

108

281522

0

0

27350

51196

-39323

320745

273

254.154

231

 

User, 09/18/12,
MUST LOOK AT THE NEGATIVE SIGNS FOR SUBTRACTION.
Page 109: Accounting and Finance

Calculate the amount allocated to cost of goods sold under each method.

FIFO$

Average-cost$

LIFO$

Click here if you would like to Show Work for this question

AE8-7

Your answer is correct.

   

Gwynne Supply Co. has the following transactions related to notes receivable during the last 2 months of the year.

Nov. 1 Loaned $64,950 cash to B. Akey on a 1-year, 8% note.Dec. 11 Sold goods to R. P. Mayrl, Inc., receiving a $5,500, 90-day, 7% note.Dec. 16 Received an $12,800, 6-month, 9% note to settle an open account from M. Colvin.Dec. 31 Accrued interest revenue on all notes receivable.

Journalize the transactions for Gwynne Supply Co. (Round computations and final answers to 0 decimal answers, e.g. 250.)

Date Account/Description Debit Credit

Nov. 1  

           

 

Dec. 11  

          

 

Dec. 16  

           

 

Dec. 31  

           

 

AE8-5 (a,b)

Your answer is correct.

   

109

Look at Scratch Work for Chapter 6 on back page.

1143

1161.846

1185

Notes receivable 64950

Cash 64950

Notes receivable 5500

Sales 5500

Notes receivable 12800

Accounts receivable 12800

Interest receivable 935

Interest revenue 935

User, 09/18/12,
64,950 x 8% x 2/12 (Nov. to Dec. is 2 months/ months in a year) = 866 5,500 x 7% x 20/360 (Subtracted Dec. 31 – Dec. 11 = 20 days/360 days) = 2112,800 x 9% x 0.5/12 (6 months of a year needs to be represented as 0.5)= 48Total = 935
Page 110: Accounting and Finance

Parry Company has accounts receivable of $117,500 at March 31, 2012. An analysis of the accounts shows these amounts.

Balance, March 31Month of Sale 2012 2011March $69,400   $75,000  February 20,300   8,000  December and January 18,000   2,400  November and October 9,800   1,100  

$117,500   $86,500  

Credit terms are 2/10, n/30. At March 31, 2012, there is a $2,200 credit balance in Allowance for Doubtful Accounts prior to adjustment. The company uses the percentage of receivables basis for estimating uncollectible accounts. The company's estimates of bad debts are as follows.

Estimated PercentageAge of Accounts UncollectibleCurrent 3%1-30 days past due 8  31-90 days past due 31  Over 90 days past due 55  (a) Determine the total estimated

uncollectibles.    

$(b) Prepare the adjusting entry at March 31, 2012, to record bad debts expense.

DateAccount/Description Debit Credit

Mar. 31  

           

 

AE8-3

Your answer is correct.

   At the beginning of the current period, Engseth Corp. had balances in Accounts Receivable of $198,840 and in Allowance for Doubtful Accounts of $9,612 (credit). During the period, it had net credit sales of $801,245 and collections of $651,434. It wrote off as uncollectible accounts receivable of $5,852. However, a $3,922 account previously written off as uncollectible was recovered before the end of the current period. Uncollectible accounts are estimated to total $21,417 at the end of the period.

(a) Prepare the entries to record sales and collections during the period.Account/Description Debit Credit

 

110

Multiple percentages by month of sales 2012 respectively. Add to get total estimated uncollectibles.

14676

Bad debts expense 12476

 

 

 

Allow ance for doubtful accounts 12476

Accounts receivable 801245

 

User, 09/18/12,
Subtract 2,200 credit balance from total estimated uncollectibles = bad debts expense.
Page 111: Accounting and Finance

          

(To record the sales.)    

 

           

(To record the collections.)    (b) Prepare the entry to record the write-off of uncollectible accounts during the period.

Account/Description Debit Credit

 

           

(c) Prepare the entries to record the recovery of the uncollectible account during the period.Account/Description Debit Credit

 

          

(To reinstate the account receivable.)    

 

           

(To record the collection.)    (d) Prepare the entry to record bad debts expense for the period.

Account/Description Debit Credit

 

           

(e) Determine the ending balances in Accounts Receivable and Allowance for Doubtful Accounts.Accounts receivable

$

Allowance for doubtful accounts$

(f) What is the net realizable value of the receivables at the end of the period?     $ 321,382

ABE9-13

Your answer is correct.

   

Alan Chemicals Company acquires a delivery truck at a cost of $37,570 on January 1, 2012. The truck is expected to have a salvage value of $4,602 at the end of its 5-year useful life. Assuming the declining-balance depreciation rate is double the straight-line rate, compute annual depreciation for the first and second years under the declining-balance method. (Round your computations and answers to 0 decimal places, e.g. 12,550.)

First year depreciation$

111

 

Sales 801245

Cash 651434

Accounts receivable 651434

Allow ance for doubtful accounts 5852

Accounts receivable 5852

Accounts receivable 3922

Allow ance for doubtful accounts 3922

Cash 3922

Accounts receivable 3922

Bad debts expense 13735

Allow ance for doubtful accounts 13735

342799

21417

15028

 

 

 

User, 09/18/12,
37,570 x .40 (100% / 5) = 15,028.
User, 09/18/12,
342,799 – 21,417 = 321,382
User, 09/18/12,
198,840 + 801,245 = 1,000,085 – 651,434 = 348,651 – 5,852 = 342,799. Numbers for computations are listed in the paragraph.
User, 09/18/12,
9,612 – 5,852 = 3,760 + 3,922 = 7,682. Subtract 21,417 – 7,682 = 13,735. Numbers for computations are listed in the paragraph.
Page 112: Accounting and Finance

Second year depreciation$

Click here if you would like to Show Work for this question

ABE9-3

Your answer is correct.

   Alan Chemicals Company acquires a delivery truck at a cost of $22,584 on January 1, 2012. The truck is expected to have a salvage value of $4,044 at the end of its 4-year useful life. Compute annual depreciation for the first and second years using the straight-line method. (Round your answers to 0 decimal places, e.g. 2,550.)

First year depreciation$

Second year depreciation$

Click here if you would like to Show Work for this question

ABE9-14

Your answer is correct.

   

DriveUp Taxi Service uses the units-of-activity method in computing depreciation on its taxicabs. Each cab is expected to be driven 150,000 miles.  Taxi 10 cost $23,315 and is expected to have a salvage value of $698.  Taxi 10 was driven 27,230 miles in 2011 and 19,119 miles in 2012. Compute the depreciation for each year. (Round your per unit cost to 2 decimal, e.g. 0.25 places before computing depreciation. Round your answers to 0 decimal places, e.g. 12,550.)

First year depreciation$

Second year depreciation$

ABE9-7

Your answer is correct.

   Prepare journal entries to record these transactions: (a) Benton Company retires its delivery equipment, which cost $45,021. Accumulated depreciation is also $45,021 on this delivery equipment. No salvage value is received. (b) Assume the same information as in part (a), except that accumulated depreciation for Benton Company is $39,194 instead of $45,021. (List multiple debit/credit entries from largest

112

9016.80

4635

4635

4085

2868

User, 09/18/12,
23,315 – 698 = 22,617 / 150,000 miles = .15. Next, 19,119 miles x .15 = 2868
User, 09/18/12,
23,315 – 698 = 22,617 / 150,000 miles = .15. Next, 27,230 miles x .15 = 4085
User, 09/18/12,
22,584 – 4,044 = 18,540 / 4 = 4635
User, 09/18/12,
37,570 – 15,028 = 22,542 x .40 = 9,016.80
Page 113: Accounting and Finance

to smallest amount, e.g. 10, 5, 2.)

  Account/Description Debit Credit

(a)  

       

 

(b)  

   

       

 

Click here if you would like to Show Work for this question

113

Accum. depr.-delivery equipment 45021

Delivery equipment 45021

Accum. depr.-delivery equipment 39194

Loss on disposal 5827

Delivery equipment 45021

User, 09/18/12,
Subtract Delivery equipment from Accum.depr. – delivery equipment.